+7 495 120-13-73 | 8 800 500-97-74

(для регионов бесплатно)

Содержание

лабораторная работа 32

Рассмотрим классическую теорию.

Под действием внешнего электрического поля  электроны  будут совершать упорядоченное движение (см. рис. 3), т.к. на них  действует  электрическая сила Fе = еЕ , кроме того, на электрон со стороны поля кристаллической решетки  действует сила сопротивления Fс = -. Поэтому общая сила, действующая на электрон, равна

F = еЕ –.                                       (2)

 

По второму закону Ньютона

mа = еЕ –.                                    (3)

 

Дрейфовая скорость  будет увеличиваться до тех пор, пока Fе = Fс, а = 0, т.е. еЕ = , отсюда

u =.                                            (4).

 

Величина, равная b = , называется подвижностью электрона.

 

Смысл коэффициента следующий. Пусть Е = 0, тогда (3) запишется в

виде mа = –.Так как ускорение электрона определяется по формуле: 

 

, то

 

  ,      или      .             (5)

 

Из (5) следует, что u=. Если , то t = τ и  это время, за ко-

торое электрон  уменьшает дрейфовую скорость в  раз. За это время он пробегает расстояние, которое называется транспортным:

L = ,                                           (6)

где γ – число столкновений (рассеяния) электронов с дефектами; – средняя длина свободного пробега электрона между двумя последовательными столкновениями. За  время τ электрон испытывает γ число столкновений с дефектами кристаллической решетки и пробегает транспортное расстояние L.

С другой стороны, транспортное расстояние равно L=, так как электрон движется со средней скоростью.

С  учетом (6) получим

 или .                          (7)

 

Подставляя (7) в (4), найдем

u = .                                       (8)

 

Плотность тока равна

j = u,                                                          (9)

 

где n – концентрация электронов.

Подставляя (8) в (9), получим

j =.                                    (10)

 

Закон Ома в дифференциальной форме

j =.                                                (11)

 

Тогда с учетом (10) и (11) имеем

 и .                    (12)

 

Полученная, с точки зрения квантовой теории, формула удельного сопротивления  ρ имеет такой же вид, что и формулы (12). Но вместо  m, , , рассмотренные в классической теории, в квантовой теории вводятся: эффективная масса , фермиевская скорость   и длина свободного пробега . Учитывая эти изменения, формула (12) приобретет следующий вид (13):

.                                     (13)

 

Квантово-механические расчеты показывают, что при низких температурах число столкновений , а . Тогда удельное сопротивление ρ пропорционально и сопротивление металлов обусловлено рассеянием электронов проводимости на дефектах и ионах кристаллической решетки.

При высоких температурах сопротивление в основном обусловлено рассеянием электронов на тепловых ионах кристаллической решетки (фононах), а сопротивление, обусловленное рассеянием электронов на дефектах, пренебрежимо мало. Поэтому при высоких температурах  не зависит от

Т, а . Тогда .

Расчет показывает, что при высоких температурах удельное сопротивление зависит от температуры:

 ,                                              (14)

где α – температурный коэффициент сопротивления;  – удельное сопротивление при температуре 0 °С; t – температура по шкале Цельсия.

Температурный коэффициент α равен относительному изменению удельного сопротивления при изменении температуры проводника на 1 градус:

, .

 

Для металлических изотропных проводников коэффициент  почти не зависит от температуры и примерно равен . Так как сопротивление металлических проводников зависит от  [см. формулу (1)], т.е.

R~, то с учетом (14) сопротивление можно представить в виде

R = R0(1+).                                      (15)

 

Откуда можно найти :

  ,                                      (16)

 

где R­0 – сопротивление проводника при t = 0°С.

 

Для металлических проводников , т.е. с увеличением температуры сопротивление увеличивается. Поэтому эти проводники называются проводниками I рода. Для электролитов, графита и других , т.е. с увеличением температуры сопротивление проводника уменьшается. Они называются проводниками II рода.

 

Ход работы

 

Схема установки приведена на рис. 4. Исследуемый проводник помещается в колбу, заполненную непроводящей жидкостью (масло, глицерин, дистиллированная вода). Температура измеряется термометром.

Для равномерного нагревания жидкость перемешивается мешалкой. Исследуемый проводник R подключается в качестве неизвестного сопротивления к клеммам измерительного моста. Измеряют сопротивление исследуемого проводника при комнатной температуре.

Включают нагреватель и непрерывно перемешивают жидкость мешалкой.

Зависимость сопротивления от температуры

Электрический ток в металлах. Зависимость сопротивления металлов от температуры. Сверхпроводимость. Сопротивление проводников изменяется при изменении их температуры. С увеличением температуры сопротивление металлических проводников увеличивается, а сопротивление угля, растворов и расплавов солей и кислот уменьшается.

В металле электроны движутся в электрическом поле ионов, совершающих тепловые колебания в узлах кристаллической решетки. Сопротивление металлических проводников возникает из-за столкновения свободных электронов с ионами, при которых энергия поступательного движения электронов передаётся ионам.  Чем выше температура, тем больше амплитуда колебаний ионов и тем большая часть энергии электронов передаётся ионам. Поэтому  с ростом температуры электрическое сопротивление должно увеличиваться. У большинства металлов при не слишком низких температурах удельное сопротивление примерно пропорционально абсолютной температуре:

ρ = b T ,

где ρ — удельное сопротивление. Представим коэффициент пропорциональности в виде b =  ρ0/T0, где ρ0 — удельное сопротивление при температуре T0 =273ºК
(т. е. при 0º С). Тогда

Перейдем от абсолютной температуры к температуре по шкале Цельсия, заменив T на T0+t:

 .

где α = 1/T0 — величина, называемая температурным коэффициентом сопротивления. Коэффициент α можно представить в виде

где ρt — значение ρ при температуре t=0ºС. Значение α =1/T0 является приближенным. На практике коэффициент α определяют по формуле (2), используя экспериментальные значения,  полученные для 

ρ и ρ0.  Некоторые металлы и сплавы при охлаждении до температуры  низких температур сопротивление скачком полностью исчезает. Это явление называется сверхпроводимостью, а вещество в таком состоянии — сверхпроводником. Иная, чем у металлов, зависимость сопротивления от температуры у полупроводников. 

Металлы являются хорошими проводниками благодаря тому, что электроны внешних оболочек их атомов могут свободно перемещаться внутри металла, и этих «свободных» электронов в металлах очень много. Это электроны проводимости. В полупроводниках  концентрация свободных электронов мала и сильно зависит от температуры. Сопротивление полупроводников при нагревании убывает, т. к. с увеличением температуры увеличивается концентрация носителей заряда, концентрация  свободных электронов и дырок.  К полупроводникам относятся  кремний, германий, селен, теллур, бор, мышьяк, фосфор и некоторые другие элементы и соединения.

Полупроводниковые приборы, в которых используется сильная зависимость сопротивления полупроводников от температуры, называются термисторами или термосопротивлениями.  Приборы, действие которых основано на зависимости сопротивления полупроводников от освещенности, называются фотосопротивлениями.

Распространённые ошибки при термоэлектрических расчётах

При моделировании различных электротехнических приборов важную роль играет правильный расчёт электромагнитного нагрева материалов, электропроводность и теплопроводность которых нелинейно зависят от температуры. При моделировании таких нелинейностей даже у опытных инженеров могут возникать некоторые трудности и неожиданные результаты расчетов при комплексных сочетаниях нелинейных свойств материалов, граничных условий и геометрии. Давайте на простом примере разберём, почему это происходит.

Закон Ома и резистивный нагрев

Одним из первых физических законов, с которым сталкиваемся и который используем мы как инженеры, является закон Ома: Ток, протекающий через устройство равен приложенному напряжению (разности потенциалов), делённому на электрическое сопротивление или I = V/Re, где Re — электрическое сопротивление, которое является функцией от геометрии и электропроводности материала.

Далее мы узнаём о рассеиваемой мощности, которая равна произведению приложенного напряжения и тока, или Q = IV, что можно также записать, как Q = I2Re или Q = V2/Re. Немного позже мы сталкиваемся с такими свойствами, как теплопроводность и эквивалентное термическое сопротивление устройства, Rt, которые позволяют нам рассчитать повышение температуры устройства (относительно условий окружающей среды) по формуле ΔT = QRt.

Теперь мы можем определить абсолютное значение температуры устройства по формуле T = T_ {ambient} + QR_t. С этого момента мы и начнём наше обсуждение. Рассмотрим полностью дискретную модель. Да, пример в данной статье настолько прост, что нам даже не придётся использовать программное обеспечение COMSOL Multiphysics® для этой части. Итак, у нас есть модель с сосредоточенными параметрами, электрическое сопротивление которого равно Re = 1 Ω, а тепловое сопротивление — Rt = 1 K/W. 2 R_e R_t.

Температуру окружающей среды примем равной 300 K или 27°C, что является примерной комнатной температурой. Теперь давайте рассчитаем температуру нашего устройства, как функцию от напряжения (от 0 до 10 В) и тока (от 0 до 10 А), как показано на рисунке ниже. Неудивительно, что мы видим квадратичную зависимость.


Зависимость температуры устройства от приложенного напряжения (слева) и протекающего тока (справа) при постоянных свойствах материала.

Можно предположить, что возможно использовать кривую для расчёта более широкого диапазона рабочих параметров. Поставим задачу нагреть устройство до критической температуры, при которой материал начнёт плавиться и испаряться. Предположим, что он начнёт испаряться при температуре 700 K (427°C). Основываясь на графиках выше и проведя простые математические вычисления можно рассчитать, что максимальное напряжение будет равно 20 В, а ток — 20 А, однако это совершенно неправильно!

Учет нелинейных свойств материала для сосредоточенной модели

На данном этапе вы можете заметить, что допущена ошибка: Электрическое сопротивление непостоянно и зависит от температуры. e_0)) R_t

Эти уравнения уже немного сложнее (первое — квадратичное уравнение относительно T), но их ещё можно решить вручную. Графики зависимости температуры от приложенного напряжения и тока показаны ниже.


Зависимости температуры устройства от приложенного напряжения (слева) и протекающего тока (справа) для случая зависимости электрического сопротивления от температуры.

Для варианта с напряжением мы видим, что при увеличении температуры, сопротивление также возрастает. Так как в этом случае сопротивление в уравнении для температуры находится в знаменателе, с его увеличением прирост температуры будет уменьшаться: мы видим, что график температуры лежит ниже, чем для случая с постоянным удельным сопротивлением. Если рассматривать вариант с постоянным током, то зависящее от температуры сопротивление будет в числителе.

По мере увеличения тока резистивный нагрев будет, наоборот, больше, чем в первом случае. На данном этапе мы опять-таки можем вычислить максимальное рабочее напряжение и ток, однако, вы, вероятно, уже видите вторую ошибку, которую мы допустили: необходимо также учитывать температурную зависимость теплового сопротивления. t_0 = 300 K), можно построить зависимости температуры устройства, как показано ниже.


Зависимости температуры устройства от приложенного напряжения (слева) и протекающего тока (справа) для случая зависимости электрического и термического сопротивлений от температуры.

Заметим, что для варианта с током температура возрастает асимптотически. Поскольку электрическое и тепловое сопротивление увеличиваются с повышением температуры, температура устройства возрастает очень резко по мере увеличения тока. При повышении температуры до бесконечности, задача становится нерешаемой. На самом деле, это полностью ожидаемо. Так работает предохранитель в вашем автомобиле. Если решать данную задачу в COMSOL Multiphysics, то можно использовать исследование во временной области (введя термальную массу как функцию от от плотности материала и теплоемкости), с помощью которого можно рассчитать время, в течение которого произойдет отказ устройства.

Для варианта с напряжением всё намного проще. Здесь мы также видим вполне предсказуемое поведение системы. С увеличением сопротивления температура растёт быстрее, чем в предыдущем случае. Однако заметьте, что относительно самого первого случая с постоянным сопротивлением, температура всё же ниже. Иногда это может запутать, но просто отметьте для себя, что одна из нелинейностей приводит к уменьшению температуры, а другая — к увеличению. В целом, если составлять более комплексную модель (например, которую можно сделать и рассчитать в COMSOL Multiphysics), то сложно предугадать, какая из нелинейностей будет преобладать.

Какую ещё ошибку можно допустить на этом этапе? В данном случае мы использовали положительный температурный коэффициент теплового сопротивления. Это справедливо для большинства металлов, однако для изоляторов, к примеру для стекла, этот коэффициент будет противоположным. Обычно термическое сопротивление всего устройства в основном зависит как раз от изоляторов, а не от электропроводящих доменов. Кроме того, при расчёте термического сопротивления устройства необходимо учитывать естественное охлаждение. То есть термического сопротивление также будет зависеть от эффектов свободной конвекции (которая возрастает с разницей температур) и излучения (которое пропорционально четвёртой степени разницы температур). Теперь давайте закончим рассмотрение нашей относительно простой задачи и изменим знак температурного коэффициента, αt = 1/400 K, а затем также сравним два варианта — с питающим напряжением (от 0 до 100 В) и протекающим током (от 0 до 100 А).


Зависимости температуры устройства от приложенного напряжения (розовым) и протекающего тока (голубым) при отрицательном температурном коэффициенте термического сопротивления.

Мы видим, что результаты теперь совершенно другие. Обратите внимание, что в обоих случаях при низких значениях сопротивления кривые возрастают квадратично, а при увеличении сопротивления они уже имеют менее резкий характер, так как термическое сопротивление уменьшается. Тангенс угла наклона всегда положителен, но его величина постепенно уменьшается. В случае с током, кривая начинает асимптотически приближаться к значению T = 700 K, а в варианте с напряжением это значение меньше.

Это достаточно важный результат, и он позволяет обратить внимание на ещё одну распространённую ошибку. Нелинейные модели материалов, которые мы здесь используем для электрического и термического сопротивлений, являются приближёнными. Они становятся неэффективными при значениях температур около 700 К. Если мы знаем, что устройство будет работать как раз в этом режиме, то нужно найти более сложную материальную модель. Несмотря на то, что существующие модели позволяли получить какой-то результат, всегда стоит проверять адекватность расчёта на действующей рабочей температуре. Конечно, если наши условия эксплуатации далеки от таких температур, можно воспользоваться линеаризованной моделью резистивности (одной из встроенных материальных моделей в COMSOL Multiphysics). Тогда наша модель будет корректной.

Подводя некоторые итоги, мы видим, что температура имеет очень сложную зависимость от питающего напряжения и тока. При рассмотрении нелинейных материалов температура может быть выше или ниже, чем при постоянных свойствах, угол наклона температурной кривой может быть как довольно резким, так и плавным, в зависимости от условий работы.

Запутали ли вас результаты в последнем варианте ещё больше? Что, если мы вернёмся к выражению для сопротивления и изменим один из коэффициентов? У некоторых материалов отрицательный знак температурных коэффициентов для электрического и термического сопротивлений. Что, если бы мы использовали более комплексные нелинейности? Вы все также будете пытаться предсказать ожидаемую температуру на основании сосредоточенной модели, или вы бы скорее положились на полноценный детальный расчёт?

Выводы о распространённых ошибках при электротермических расчётах

Как насчёт случая реального устройства? В нём будет несколько материалов, различные зависимости электро- и теплопроводностей как функции температуры, а также и сложная геометрия. Какой решатель при моделировании вы бы выбрали: стационарный или временной, чтобы узнать, сколько времени потребуется для повышения температуры? Скорее всего, в модели также будут нелинейные граничные условия такие, как условие излучения и свободная конвекция, которые неправильно будет аппроксимировать только одним приближённым тепловым сопротивлением. Что же в таком случае можно ожидать? Да практически всё! И как же рассчитывать такие сложные задачи? Конечно же с помощью COMSOL Multiphysics!

Следующий шаг

Узнайте, как COMSOL Multiphysics может помочь вам в мультифизическом моделировании при решении поставленных задач. Не стесняйтесь написать нам и задать все интересующие вопросы!

Физика — 11

ИССЛЕДОВАНИЕ1

От чего зависит удельное сопротивление металлического проводника?

Задача 1. Основная часть электрического нагревателя состоит из проводника, изготовленного из сплава специально подобранных металлов. Какие проводники выбирают для изготовления нагревателя:
a) обладающие большим удельным сопротивлением или малым?
b) большей длины или меньшей?
c) с большой площадью поперечного сечения или меньшей?
Обсуждение результата:

• От чего зависят сопротивление и удельное сопротивление металлического проводника?

Закон Ома для участка цепи. Существование разности потенциалов на концах проводника — необходимое условие возникновения электрического тока в нем. Разность потенциалов (электрическое напряжение) на концах произвольного участка цепи численно равна работе по перенесению заряда 1 Кл по этому участку:

U = A
q. (1).

Сила тока, проходящего через участок цепи, зависит от напряжения на концах участка. Эту зависимость в 1827 году экспериментально установил немецкий ученый Георг Ом (1787-1854), и поэтому её называют законом Ома:

• Ста тока на определенном участке цепи прямо пропорциональна напряжению на её концах и обратно пропорциональна сопротивлению этого участка:

I = U
R. (2)

Зависимость силы тока в проводнике от напряжения на его концах называют вольт-амперной характеристикой (ВАХ) проводника. Для металлических проводников ВАХ представляет собой прямую, проходящую через начало координат (a).

• Произведение сопротивления участка цепи на силу проходящего по нему тока называют падением напряжения на этом участке (проводнике):

U = IR. (3)

Здесь R — сопротивление проводника, является одной из его основных характеристик. Сопротивление данного проводника имеет определенное значение, не зависящее от силы тока в нем и напряжения на его концах.

• Сопротивление металлического проводника зависит от материала, из которого он изготовлен, его геометрических параметров и температуры:

R = ρ ι
S. (4)

Сопротивление проводника измеряют прибором, называемым омметром (b). Сопротивление на устройстве обозначается символом Ω (омега). Для этого достаточно прикоснуться соединительными проводами прибора к концам металлического проводника, сопротивление которого измеряется.

Зависимость сопротивления металлов от температуры. Электрическое сопротивление — мера противодействия проводника возникновению в нем электрического тока. Согласно классической электронной теории причиной сопротивления металлического проводника являются столкновения свободных электронов в нем с положительными ионами, совершающими колебательное движение в узлах его кристаллической решетки.

Как зависит сопротивление от температуры. Зависимость сопротивления проводника от температуры

В своей практической деятельности каждый электрик встречается с разными условиями прохождения носителей зарядов в металлах, полупроводниках, газах и жидкостях. На величину тока влияет электрическое сопротивление, которое различным образом изменяется под влиянием окружающей среды.

Одним из таких факторов является температурное воздействие. Поскольку оно значительно изменяет условия протекания тока, то учитывается конструкторами в производстве электрооборудования. Электротехнический персонал, участвующий в обслуживании и эксплуатации электроустановок, обязан грамотно использовать эти особенности в практической работе.

Влияние температуры на электрическое сопротивление металлов

В школьном курсе физики предлагается провести такой опыт: взять амперметр, батарейку, отрезок проволоки, соединительные провода и горелку. Вместо амперметра с батарейкой можно подключить омметр или использовать его режим в мультиметре.

Теперь поднесем пламя горелки к проволоке и станем ее нагревать. Если смотреть на амперметр, то будет видно, что стрелка станет перемещаться влево и достигнет положения, отмеченного красным цветом.

Результат опыта демонстрирует, что при нагревании металлов их проводимость уменьшается, а сопротивление возрастает.

Математическое обоснование этого явления приведено формулами прямо на картинке. В нижнем выражении хорошо видно, что электрическое сопротивление «R» металлического проводника прямо пропорционально его температуре «Т» и зависит еще от нескольких параметров.

Как нагрев металлов ограничивает электрический ток на практике

Лампы накаливания

Ежедневно при включении освещения мы встречаемся с проявлением этого свойства у ламп накаливания. Проведем несложные измерения на лампочке с мощностью 60 ватт.

Самым простым омметром, питающемся от низковольтной батарейки 4,5 V, замерим сопротивление между контактами цоколя и увидим значение 59 Ом. Этой величиной обладает нить накала в холодном состоянии.

Вкрутим лампочку в патрон и через амперметр подключим к ней напряжение домашней сети 220 вольт. Стрелка амперметра покажет 0,273 ампера. По определим сопротивление нити в нагретом состоянии. Оно составит 896 Ом и превысит предыдущее показание омметра в 15,2 раза.

Такое превышение предохраняет металл тела накала от перегорания и разрушения, обеспечивая его длительную работоспособность под напряжением.

Переходные процессы при включении

При работе нити накала на ней создается тепловой баланс между нагревом от проходящего электрического тока и отводом части тепла в окружающую среду. Но, на первоначальном этапе включения при подаче напряжения возникают переходные процессы, создающие бросок тока, который может привести к перегоранию нити.

Переходные процессы протекают за короткое время и вызваны тем, что скорость возрастания электрического сопротивления от нагрева металла не успевает за увеличением тока. После их окончания устанавливается рабочий режим.

Во время длительного свечения лампы постепенно толщина ее нити доходит до критического состояния, которое приводит к перегоранию. Чаще всего этот момент возникает при очередном новом включении.

Для продления ресурса лампы различными способами уменьшают этот бросок тока, используя:

1. устройства, обеспечивающие плавную подачу и снятие напряжения;

2. схемы последовательного подключения к нити накала резисторов, полупроводников или терморезисторов (термисторов).

Пример одного из способов ограничения пускового тока для автомобильных светильников показан на картинке ниже.

Здесь ток на лампочку подается после включения тумблера SA через предохранитель FU и ограничивается резистором R, у которого номинал подбирается так, чтобы бросок тока во время переходных процессов не превышал номинальное значение.

При нагреве нити накала ее сопротивление возрастает, что ведет к увеличению разности потенциалов на ее контактах и параллельно подключенной обмотке реле KL1. Когда напряжение достигнет величины уставки реле, то нормально открытый контакт KL1 замкнется и зашунтирует резистор. Через лампочку начнет протекать рабочий ток уже установившегося режима.

Влияние температуры металла на его электрическое сопротивление используется в работе измерительных приборов. Их называют .

Их чувствительный элемент выполняют тонкой проволочкой из металла, сопротивление которой тщательно замерено при определенных температурах. Эту нить монтируют в корпусе со стабильными термическими свойствами и закрывают защитным чехлом. Созданная конструкция помещается в среду, температуру которой необходимо постоянно контролировать.

На выводы чувствительного элемента монтируются провода электрической схемы, которыми подключается цепь замера сопротивления. Его величина пересчитывается в значения температуры на основе ранее произведенной калибровки прибора.

Бареттер — стабилизатор тока

Так называют прибор, состоящий из стеклянного герметичного баллона с газообразным водородом и металлической проволочной спиралью из железа, вольфрама или платины. Эта конструкция по внешнему виду напоминает лампочку накаливания, но она обладает специфической вольт-амперной нелинейной характеристикой.

На ВАХ в определенном ее диапазоне образуется рабочая зона, которая не зависит от колебаний приложенного на тело накала напряжения. На этом участке бареттер хорошо компенсирует пульсации питания и работает в качестве стабилизатора тока на подключенной последовательно к нему нагрузке.

Работа бареттера основана на свойстве тепловой инерции тела накала, которая обеспечивается маленьким сечением нити и высокой теплопроводностью окружающего ее водорода. За счет этого при снижении напряжения на приборе ускоряется отвод тепла с его нити.

Это основное отличие бареттера от осветительных ламп накаливания, в которых для поддержания яркости свечения стремятся уменьшить конвективные потери тепла с нити.

Сверхпроводимость

В обычных условиях среды при охлаждении металлического проводника происходит уменьшение его электрического сопротивления.

При достижении критической температуры, близкой к нулю градусов по системе измерения Кельвина, происходит резкое падение сопротивления до нулевого значения. На правой картинке показана такая зависимость для ртути.

Это явление, названное сверхпроводимостью, считается перспективной областью для исследований с целью создания материалов, способных значительно снизить потери электроэнергии при ее передаче на огромные расстояния.

Однако, продолжающиеся изучения сверхпроводимости выявили ряд закономерностей, когда на электрическое сопротивление металла, находящегося в области критических температур, влияют другие факторы. В частности, при прохождении переменного тока с повышением частоты его колебаний возникает сопротивление, величина которого доходит до диапазона обычных значений у гармоник с периодом световых волн.

Влияние температуры на электрическое сопротивление/проводимость газов

Газы и обычный воздух являются диэлектриками и не проводят электрический ток. Для его образования нужны носители зарядов, которыми выступают ионы, образующиеся в результате воздействия внешних факторов.

Нагрев способен вызвать ионизацию и движение ионов от одного полюса среды к другому. Убедиться в этом можно на примере простого опыта. Возьмем то же оборудование, которым пользовались для определения влияния нагрева на сопротивление металлического проводника, только вместо проволоки к проводам подключим две металлические пластины, разделенные воздушным пространством.

Подсоединенный к схеме амперметр покажет отсутствие тока. Если между пластинами поместить пламя горелки, то стрелка прибора отклонится от нулевого значения и покажет величину проходящего через газовую среду тока.

Таким образом установили, что в газах при нагревании происходит ионизация, приводящая к движению электрически заряженных частиц и снижению сопротивления среды.

На значении тока сказывается мощность внешнего приложенного источника напряжения и разность потенциалов между его контактами. Она способна при больших значениях пробить изоляционный слой газов. Характерным проявлением подобного случая в природе является естественный разряд молнии во время грозы.

Примерный вид вольт-амперной характеристики протекания тока в газах показан на графике.

На начальном этапе под действие температуры и разности потенциалов наблюдается рост ионизации и прохождение тока примерно по линейному закону. Затем кривая приобретает горизонтальное направление, когда увеличение напряжения не вызывает рост тока.

Третий этап пробоя наступает тогда, когда высокая энергия приложенного поля так разгоняет ионы, что они начинают соударяться с нейтральными молекулами, массово образуя из них новые носители зарядов. В результате ток резко возрастает, образуя пробой диэлектрического слоя.

Практическое использование проводимости газов

Явление протекания тока через газы используется в радиоэлектронных лампах и люминесцентных светильниках.

Для этого внутри герметичного стеклянного баллона с инертным газом располагают два электрода:

1. анод;

2. катод.

У люминесцентной лампы они выполнены в виде нитей накала, которые разогреваются при включении для создания термоэлектронной эмиссии. Внутренняя поверхность колбы покрыта слоем люминофора. Он излучает видимый нами спектр света, образующийся при инфракрасном облучении, исходящем от паров ртути, бомбардируемых потоком электронов.

Ток газового разряда возникает при приложении напряжения определенной величины между электродами, расположенными по разным концам колбы.

Когда одна из нитей накала перегорит, то на этом электроде нарушится электронная эмиссия и лампа гореть не будет. Однако, если увеличить разность потенциалов между катодом и анодом, то снова возникнет газовый разряд внутри колбы и свечение люминофора возобновится.

Это позволяет использовать светодиодные колбы с нарушенными нитями накала и продлять их ресурс работы. Только следует учитывать, что при этом в несколько раз надо поднять на ней напряжение, А это значительно повышает потребляемую мощность и риски безопасного использования.

Влияние температуры на электрическое сопротивление жидкостей

Прохождение тока в жидкостях создается в основном за счет движения катионов и анионов под действием приложенного извне электрического поля. Лишь незначительную часть проводимости обеспечивают электроны.

Влияние температуры на величину электрического сопротивления жидкого электролита описывается формулой, приведенной на картинке. Поскольку в ней значение температурного коэффициента α всегда отрицательно, то с увеличением нагрева проводимость возрастает, а сопротивление падает так, как показано на графике.

Это явление необходимо учитывать при зарядке жидкостных автомобильных (и не только) аккумуляторных батарей.

Влияние температуры на электрическое сопротивление полупроводников

Изменение свойств полупроводниковых материалов под воздействием температуры позволило использовать их в качестве:

    термических сопротивлений;

    термоэлементов;

    холодильников;

    нагревателей.

Терморезисторы

Таким названием обозначают полупроводниковые приборы, изменяющие свое электрическое сопротивление под влиянием тепла. Их значительно выше, чем у металлов.

Величина ТКС у полупроводников может иметь положительное или отрицательное значение. По этому параметру их разделяют на позитивные «РТС» и негативные «NTC» термисторы. Они обладают различными характеристиками.

Для работы терморезистора выбирают одну из точек на его вольт-амперной характеристике:

    линейный участок применяют для контроля температуры либо компенсации изменяющихся токов или напряжений;

    нисходящая ветвь ВАХ у элементов с ТКС

Применение релейного терморезистора удобно при контроле или измерениях процессов электромагнитных излучений, происходящих на сверхвысоких частотах. Это обеспечило их использование в системах:

1. контроля тепла;

2. пожарной сигнализации;

3. регулирования расхода сыпучих сред и жидкостей.

Кремниевые терморезисторы с маленьким ТКС>0 используют в системах охлаждения и стабилизации температуры транзисторов.

Термоэлементы

Эти полупроводники работают на основе явления Зеебека: при нагреве спаянного места двух разрозненных металлов на стыке замкнутой цепи возникает ЭДС. Таким способом они превращают тепловую энергию в электричество.

Конструкцию из двух таких элементов называют термопарой. Ее КПД лежит в пределах 7÷10%.

Термоэлементы используют в измерителях температур цифровых вычислительных устройств, требующих миниатюрные габариты и высокую точность показаний, а также в качестве маломощных источников тока.

Полупроводниковые нагреватели и холодильники

Они работают за счет обратного использования термоэлементов, через которые пропускают электрический ток. При этом на одном месте спая происходит его нагрев, а на противоположном — охлаждение.

Полупроводниковые спаи на основе селена, висмута, сурьмы, теллура позволяют обеспечить разность температур в термоэлементе до 60 градусов. Это позволило создать конструкцию холодильного шкафа из полупроводников с температурой в камере охлаждения до -16 градусов.

>>Физика: Зависимость сопротивления проводника от температуры

Различные вещества имеют разные удельные сопротивления (см. § 104). Зависит ли сопротивление от состояния проводника? от его температуры ? Ответ должен дать опыт.
Если пропустить ток от аккумулятора через стальную спираль, а затем начать нагревать ее в пламени горелки, то амперметр покажет уменьшение силы тока. Это означает, что с изменением температуры сопротивление проводника меняется.
Если при температуре, равной 0°С, сопротивление проводника равно R 0 , а при температуре t оно равно R , то относительное изменение сопротивления, как показывает опыт, прямо пропорционально изменению температуры t :

Коэффициент пропорциональности α называют температурным коэффициентом сопротивления . Он характеризует зависимость сопротивления вещества от температуры. Температурный коэффициент сопротивления численно равен относительному изменению сопротивления проводника при нагревании на 1 К. Для всех металлических проводников коэффициент α > 0 и незначительно меняется с изменением температуры. Если интервал изменения температуры невелик, то температурный коэффициент можно считать постоянным и равным его среднему значению на этом интервале температур. У чистых металлов α ≈ 1/273 K -1 . У растворов электролитов сопротивление с ростом температуры не увеличивается, а уменьшается . Для них α α ≈ -0,02 K -1 .
При нагревании проводника его геометрические размеры меняются незначительно. Сопротивление проводника меняется в основном за счет изменения его удельного сопротивления. Можно найти зависимость этого удельного сопротивления от температуры, если в формулу (16.1) подставить значения

. Вычисления приводят к следующему результату:

Так как α мало меняется при изменении температуры проводника, то можно считать, что удельное сопротивление проводника линейно зависит от температуры (рис.16.2 ).


Увеличение сопротивления можно объяснить тем, что при повышении температуры увеличивается амплитуда колебаний ионов в узлах кристаллической решетки, поэтому свободные электроны сталкиваются с ними чаще, теряя при этом направленность движения. Хотя коэффициент α довольно мал, учет зависимости сопротивления от температуры при расчете нагревательных приборов совершенно необходим. Так, сопротивление вольфрамовой нити лампы накаливания увеличивается при прохождении по ней тока более чем в 10 раз.
У некоторых сплавов, например у сплава меди с никелем (константан), температурный коэффициент сопротивления очень мал: α ≈ 10 -5 K -1 ; удельное сопротивление константана велико: ρ ≈ 10 -6 Ом м. Такие сплавы используют для изготовления эталонных сопротивлений и добавочных сопротивлений к измерительным приборам, т. е. в тех случаях, когда требуется, чтобы сопротивление заметно не менялось при колебаниях температуры.
Зависимость сопротивления металлов от температуры используют в термометрах сопротивления . Обычно в качестве основного рабочего элемента такого термометра берут платиновую проволоку, зависимость сопротивления которой от температуры хорошо известна. Об изменениях температуры судят по изменению сопротивления проволоки, которое можно измерить.
Такие термометры позволяют измерять очень низкие и очень высокие температуры, когда обычные жидкостные термометры непригодны.
Удельное сопротивление металлов растет линейно с увеличением температуры. У растворов электролитов оно уменьшается при увеличении температуры.

???
1. Когда электрическая лампочка потребляет большую мощность: сразу после включения ее в сеть или спустя несколько минут?
2. Если бы сопротивление спирали электроплитки не менялось с температурой, то ее длина при номинальной мощности должна быть большей или меньшей?

Г.Я.Мякишев, Б.Б.Буховцев, Н.Н.Сотский, Физика 10 класс

Содержание урока конспект урока опорный каркас презентация урока акселеративные методы интерактивные технологии Практика задачи и упражнения самопроверка практикумы, тренинги, кейсы, квесты домашние задания дискуссионные вопросы риторические вопросы от учеников Иллюстрации аудио-, видеоклипы и мультимедиа фотографии, картинки графики, таблицы, схемы юмор, анекдоты, приколы, комиксы притчи, поговорки, кроссворды, цитаты Дополнения рефераты статьи фишки для любознательных шпаргалки учебники основные и дополнительные словарь терминов прочие Совершенствование учебников и уроков исправление ошибок в учебнике обновление фрагмента в учебнике элементы новаторства на уроке замена устаревших знаний новыми Только для учителей идеальные уроки календарный план на год методические рекомендации программы обсуждения Интегрированные уроки

Если у вас есть исправления или предложения к данному уроку,

Сопротивление проводников зависит от вещества, из которого они из-готовлены, и их геометрических размеров

R = ρ . l / S,

где ρ — удельное сопротивление вещества, из которого изготовлен проводник; l —длина проводника; S — площадь попереч-ного сечения проводника.

Сопротивление проводников входит в за-кон Ома для однородного участка цепи I = U / R , из которого и может быть определено R = U / I .

Из последней формулы выходит, что со-противление проводника постоянно, посколь-ку, в соответствии с законом Ома, во сколь-ко раз увеличиваем напряжение на концах проводника, во столько же раз возрастает и сила тока в нем.

Но на практике можно наблюдать и дру-гие явления. Составим электрическую цепь, схема которой показана на рис. 7.2. В этой цепи есть источник тока с регулированным напряжением, электрическая лампа, напри-мер автомобильная, вольтметр и амперметр, показывающие напряжение на лампе и силу тока в ней. Устанавливаем на лампе напря-жение U 1 и отмечаем силу тока I 1 . Если теперь увеличить напряжение, например в 2 раза (U 2 = 2U 1), то по закону Ома и сила тока должна увеличиться в 2 раза (I 2 = 2I 1). Однако амперметр показывает силу тока значительно меньшую, чем 2I 1 . Следова-тельно, в данном случае закон Ома не вы-полняется.

Возникло несоответствие между вашими предшествующими знаниями и новым для вас фактом — закон Ома не всегда справед-лив. Такое несоответствие в науке назы-вается проблемой.

Проблема (гр. — задача, затруд-нение) — сложный теоретиче-ский или практический вопрос, требующий решения.

Можно высказывать разные предположе-ния, что является попыткой объяснить на-блюдаемое явление. Однако в ходе опыта бро-сается в глаза, что при увеличенном напря-жении лампа светится ярче, чем в первом слу-чае. Это является свидетельством того, что тем-пература спирали лампы во втором случае вы-ше, чем в первом. Возможно, именно измене-ние температуры является причиной изменения сопротивления металлической спирали лампы.

Как же можно проверить такое предпо-ложение (гипотезу)? Составляем электриче-скую цепь (рис. 7.3), в которой есть метал-лический проводник в виде спирали, на-пример пружинка от шариковой ручки, и устанавливаем в цепи ток определенной си-лы. Нагревая спираль в пламени свечи или спички, заметим:

при нагревании спирали и при постоянном напряжении сила тока в цепи уменьшается, что свидетельствует об увеличении сопротивления спирали при по-вышении ее температуры.

Тщательные исследования показывают, что сопротивление металлических проводников зависит от их температуры практически ли-нейно

R = R 0 ( 1 + α t°),

где R 0 — сопротивле-ние проводника при 0 °C или +20 °C (это удобнее для техники). График такой зави-симости представлен на рис. 7.4.

Если иметь в виду, что размеры металлов при нагревании изменяются мало, то со-ответствующую формулу можно записать и для удельного сопротивления металлических проводников

ρ = ρ 0 ( 1 + α t°).

Рассмотрим, что означает коэффициент в полученных формулах. Если при 0°C со-противление проводника R 0 , а при C со-противление его R, то относительное изме-нение сопротивления, как показывает эксперимент, (R — R 0) / R 0 = α t ° C. Материал с сайта

Коэффициент пропорциональности назы-вается температурным коэффициентом со-противления , который характеризует зави-симость сопротивления вещества от его тем-пературы.

Температурный коэффициент сопро-тивления равен относительному изменению сопротивления проводника при изменении его температуры на 1 К.

Для всех металлических проводников α > 0 и мало зависит от тем-пературы.

Почему же возрастает сопротивление ме-таллических проводников с повышением температуры? Дело в том, что при нагре-вании металла возрастает интенсивность ко-лебаний ионов кристаллической решетки и скорость хаотического движения электро-нов.

Электроны чаще сталкиваются с ионами, что и уменьшает скорость их направленного движения, которое и является электричес-ким током.

В технике зависимость сопротивления металлических проводников от температуры используется в термометрах сопротивления.

Датчик температуры (например, платиновая проволочка) устанавливается в тех точках, где необходимо измерять температуру, а его сопротивление измеряют омметром, шкала которого градуируется в единицах темпера-туры. Таких датчиков, при необходимости, может быть любое количество, а измери-тельный прибор — один.

На этой странице материал по темам:

  • График зависимости сопротивления от температуры в вакууме

  • Зависимость сопротивления от температуры для вакуума

  • Зависимость сопротивления в вакууме от температуры

  • Зависимость сопротивления металлических проводников от температуры

  • Зависимость сопротивления от температуры в вакууме график

Вопросы по этому материалу:

Каждое вещество имеет свое удельное сопротивление. Причем сопротивление будет зависеть от температуры проводника. Убедимся в этом, проведя следующий опыт.

Пропустим ток через стальную спираль. В цепи со спиралью подключим последовательно амперметр . Он покажет некоторое значение. Теперь будем нагревать спираль в пламени газовой горелки. Значение силы тока, которое покажет амперметр, уменьшится. То есть, сила тока будет зависеть от температуры проводника.

Изменение сопротивления в зависимости от температуры

Пусть при температуре 0 градусов, сопротивление проводника равняется R0, а при температуре t сопротивление равно R, тогда относительное изменение сопротивления будет прямо пропорционально изменению температуры t:

В данной формуле а — коэффициент пропорциональности, который называют еще температурным коэффициентом. Он характеризует зависимость сопротивления, которым обладает вещество, от температуры.

Температурный коэффициент сопротивления численно равен относительному изменению сопротивления проводника при нагревании его на 1 Кельвин.

Для всех металлов температурный коэффициент больше нуля. При изменениях температуры он будет незначительно меняться. Поэтому, если изменение температуры невелико, то температурный коэффициент можно считать постоянным, и равным среднему значению из этого интервала температур.

Растворы электролитов с ростом температуры сопротивление уменьшается. То есть для них температурный коэффициент будет меньше нуля.

Сопротивление проводника зависит от удельного сопротивления проводника и от размеров проводника. Так как размеры проводника при нагревании меняются незначительно, то основной составляющей изменения сопротивления проводника является удельное сопротивление.

Зависимость удельного сопротивления проводника от температуры

Попытаемся найти зависимость удельного сопротивления проводника от температуры.

Подставим в полученную выше формулу значения сопротивлений R=p*l/S R0=p0*l/S.

Получим следующую формулу:

Данная зависимость представлена на следующем рисунке.

Попробуем разобраться, почему увеличивается сопротивление

Когда мы повышаем температуру, то увеличивается амплитуда колебаний ионов в узлах кристаллической решетки. Следовательно, свободные электроны будут чаще с ними сталкиваться. При столкновении они будет терять направленность своего движения. Следовательно, сила тока будет уменьшаться.

Сопротив­ление металлов связано с тем, что электроны, движущиеся в провод­нике, взаимодействуют с ионами кристаллической решетки и теряют при этом часть энергии, которую они приобретают в электрическом поле.

Опыт показывает, что сопротив­ление металлов зави­сит от температуры. Каждое вещество можно харак­теризовать постоянной для него вели­чиной, называемой температурным коэффициентом сопротивления α . Этот коэффициент равен относитель­ному изменению удельного сопро­тивления проводника при его нагре­вании на 1 К: α =

где ρ 0 — удельное сопротивление при температуре T 0 = 273 К (0°С), ρ — удельное сопротивление при данной температуре T. Отсюда зависимость удельного сопротивления металли­ческого проводника от температуры выражается линейной функцией: ρ = ρ 0 (1+ αT).

Зависимость сопротивления от температуры выражается такой же функцией:

R = R 0 (1+ αT).

Температурные коэффициенты со­противления чистых металлов срав­нительно мало отличаются друготдруга и примерно равны 0,004 K -1 . Изменение сопротивления про­водников при изменении температу­ры приводит к тому, что их вольт-амперная характеристика не линейна. Это особенно заметно в тех слу­чаях, когда температура проводни­ков значительно изменяется, напри­мер при работе лампы накаливания. На рисунке приведена ее вольт — амперная характеристика. Как видно из рисунка, сила тока в этом случае не прямо пропорциональна напря­жению. Не следует, однако, думать, что этот вывод противоречит закону Ома. Зависимость, сформулированная в законе Ома, справедлива только при постоян­ном сопротивлении. Зависимость сопротивления ме­таллических проводников от темпе­ратуры используют в различных из­мерительных и автоматических уст­ройствах. Наиболее важным из них является термометр сопротивления . Основной частью термометра со­противления служит платиновая про­волока, намотанная на керамиче­ский каркас. Проволоку помещают в среду, температуру кото­рой нужно определить. Измеряя со­противление этой проволоки и зная ее сопротивление при t 0 = 0 °С (т. е. R 0), рассчитывают по последней формуле температуру среды.

Сверхпроводимость. Однако до конца XIX в. нельзя было прове­рить, как зависит сопротивление про­водников от температуры в области очень низких температур. Только в начале XX в. голландскому учено­му Г. Камерлинг-Оннесу удалось пре­вратить в жидкое состояние наибо­лее трудно конденсируемый газ — гелий. Температура кипения жидкого гелия равна 4,2 К. Это и дало воз­можность измерить сопротивление некоторых чистых металлов при их охлаждении до очень низкой темпе­ратуры.

В 1911г работа Камерлинг-Оннеса завершилась крупнейшим откры­тием. Исследуя сопротивление рту­ти при ее постоянном охлаждении, он обнаружил, что при температуре 4,12 К сопротивление ртути скачком падало до нуля. В даль­нейшем ему удалось это же явление наблюдать и у ряда других метал­лов при их охлаждении до темпе­ратур, близких к абсолютному нулю. Явление полной потери металлом электрического сопротивления при определенной температуре получило название сверхпроводимости.

Не все материалы могут стать сверхпроводниками, но их число до­статочно велико. Однако у многих из них было обнаружено свойство, которое значительно препятствовало их применению. Выяснилось, что у большинства чистых металлов сверхпроводимость исчезает, когда они находятся в силь­ном магнитном поле. Поэтому, когда по сверх­проводнику течет значительный ток, он создает вокруг себя магнитное поле и сверхпроводимость в нем исчезает. Всё же это препятствие оказалось преодолимым: было выяснено, что не­которые сплавы, например ниобия и циркония, ниобия и титана и др., обладают свойством сохранять свою сверхпроводимость при больших значениях силы тока. Это позволило более широко использовать сверх­проводимость.

Электронная проводимость металлов. Зависимость сопротивления от температуры

Как вы знаете, электрический ток могут проводить и твердые, и жидкие, и газообразные тела. На практике, чаще всего применяются металлические проводники. Можно привести много примеров: линии электропередач, обеспечивающие передачу энергии от различных источников тока к потребителям.

Генераторы, электронагревательные приборы и так далее. Как мы уже говорили ранее, хорошими проводниками являются некоторые растворы. Наиболее распространенный пример — это батарейка, в которой используется электролит. Примеров использования батарей и аккумуляторов тоже достаточно: они используются в автомобилях, ноутбуках, мобильных телефонах, планшетах и так далее.

Напомним, что помимо проводников, существуют такие тела, как полупроводники и диэлектрики. Как вы знаете, диэлектрики используются для изоляции проводки или электроприборов. Полупроводники представляют довольно большой интерес, поскольку их проводимостью достаточно легко управлять, а это открывает большие возможности.

Со всем выше перечисленным мы познакомимся по окончании курса физики десятого класса, и начнем с проводимости металлов.

Мы уже много раз говорили, что электрический ток — это упорядоченное движение заряженных частиц, и всегда утверждали, что в металлах носителями свободных зарядов являются электроны. Дело в том, что за этим утверждением стоят многочисленные опыты разных ученых. Мы рассмотрим несколько таких опытов.

В 1901 году, Эдуард Рикке провел следующий эксперимент: он подключил к электрической цепи металлические цилиндры, плотно прилегающие друг к другу. В центре находился алюминиевый цилиндр, а по краям — медные.

В течение приблизительно одного года через эти цилиндры протекал электрический ток. После окончания эксперимента, все три цилиндра были исследованы на предмет изменения химического состава. Выяснилось, что никаких изменений не произошло, за исключением очень незначительной диффузии. Это послужило доказательством того, что ток в металлах обусловлен именно движением электронов. Если бы в движении участвовали какие-то другие частицы (например, ионы кристаллической решетки), то это, неизбежно привело бы к изменению химического состава.

Другой опыт, был проведен в 1912 году учеными Леонидом Мандельштамом и Николаем Папалекси. К катушке, которая могла вращаться вокруг своей оси, был подключен гальванометр при помощи скользящих контактов.

При резкой остановке катушки, гальванометр регистрировал кратковременные токи. Дело в том, что при резкой остановке заряженные частицы какое-то время могли двигаться по инерции относительно проводника (то есть проволоки катушки). Поскольку сила тока характеризуется зарядом, а инерция — массой частиц, переносимый при торможении заряд пропорционален отношению заряда частиц к их массе. Из этого эксперимента было определено это соотношение, которое совпало с найденным до этого из других опытов отношением модуля заряда электрона к его массе:

Таким образом, эксперимент Мандельштама и Папалекси еще раз подтвердил, что ток в металлах обусловлен движением электронов. Поэтому, проводимость металлов называют электронной проводимостью.

Вы уже знаете, что электроны в металлах двигаются с постоянной скоростью из-за того, что взаимодействуют с ионами кристаллической решетки. Это приводит к тому, что скорость движения электронов пропорциональна напряженности электрического поля:

В свою очередь, напряженность пропорциональна напряжению. Из чего мы можем заключить, что скорость электронов в проводнике пропорциональна напряжению на концах этого проводника:

Напомним, что не так давно мы выяснили, что скорость также пропорциональна и силе тока:

Из этого мы можем сделать вывод, что 𝐼 ~ 𝑈, а это подтверждает закон Ома.

Теперь, когда мы выяснили, что электрический ток в металлах действительно обусловлен движением электронов, следует обратить внимание на одно из следствий этого явления. Электроны взаимодействуют с ионами кристаллической решетки и, тем самым нагревают проводник. Но, чем больше проводник нагревается, тем более интенсивными становятся колебания частиц проводника и тем больше они мешают движению электронов. Следовательно, в металлах существует определенная зависимость их электрического сопротивления от температуры.

Экспериментально была установлена зависимость сопротивления от температуры:

В формуле мы видим коэффициент пропорциональности α, который называется температурным коэффициентом сопротивления. Мы можем немного преобразовать выражение, описывающее зависимость сопротивления от температуры, чтобы дать определение температурному коэффициенту сопротивления:

Итак, температурный коэффициент сопротивления численно равен относительному изменению сопротивления при нагревании на 1 оС. Под относительным изменением сопротивления понимается отношение изменения сопротивления к конечному сопротивлению. Поскольку мы выяснили, что у металлов сопротивление увеличивается с увеличением температуры, можно сделать вывод, что для всех металлов коэффициент α > 0.

Вспомним, что сопротивление проводника зависит от трех величин: удельное сопротивление материала, из которого сделан проводник, площадь поперечного сечения проводника и его длина:

Поскольку геометрические размеры проводника при нагревании меняются ничтожно мало, можно сделать вывод, что изменяется удельное сопротивление:

Из полученной формулы можно сделать вывод, что удельное сопротивление металлов линейно зависит от температуры.

Эта зависимость используется в так называемых термометрах сопротивления. Термометр сопротивления представляет собой проводник, зависимость сопротивления которого от температуры хорошо известна. Чаще всего используют платиновую проволоку. Измеряя ее сопротивление можно судить о температуре. Преимущество подобного термометра заключается в том, что он пригоден для измерения температур в значительно более широком диапазоне, чем это возможно, используя жидкостные термометры.

Возникает вопрос: а что будет происходить при очень низких температурах? Этим вопросом еще в 1911 году задался Хейке Камерлинг-Оннес. В качестве опыта, он поместил ртуть в жидкий гелий и наблюдал, как постепенно уменьшается удельное сопротивление с падением температуры. Однако, когда температура опустилась до четырех целых одной десятой кельвина, сопротивление резко упало до нуля. Такое явление получило название сверхпроводимости, а температура, при которой наступает это состояние, была названа критической температурой.

Явление сверхпроводимости возникает во многих металлах при достаточно низких температурах (около 25 К). Это явление можно объяснить тем, что при таких низких температурах беспорядочное движение электронов становится очень незначительным. Иными словами, они двигаются, не соударяясь с ионами кристаллической решетки, таким образом, не замедляя своего движения и не нагревая проводник. Конечно, это объяснение существенно упрощено, но оно дает общее представление о том, как возникает явление сверхпроводимости. Тот факт, что в состоянии сверхпроводимости проводники не нагреваются, открывает большие перспективы. Если найти способ создать явление сверхпроводимости при обычных (комнатных) температурах, то можно было бы передавать электроэнергию по проводам без всяких потерь.

Сверхпроводимость используется для создания электромагнитов, которые могут создавать магнитное поле в течение длительного времени без всяких потерь энергии. Также, сверхпроводящие магниты используются в ускорителях элементарных частиц (таких как Большой Адронный Коллайдер). В 1986 году удалось создать некоторые соединения, переходящие в состояние сверхпроводимости при температурах около 100 К. На сегодняшний день, нет известных соединений, в которых бы наблюдалась сверхпроводимость при температуре выше 138 К (при нормальном давлении).

Рассмотрим еще один интереснейший эффект явления сверхпроводимости, который получил название эффекта Мейснера. Поместим два керамических цилиндра в специальную емкость и зафиксируем их.

При температуре 93 К эти цилиндры становятся сверхпроводящими. Для охлаждения можно использовать жидкий азот. Если теперь поднести к цилиндрам достаточно сильный магнит, то он зависнет над ними. Как вы знаете из курса физики девятого класса, при изменении магнитного потока через контур, возникает индукционный ток. В обычных условиях, этот ток был бы незначительным и кратковременным. Однако, в состоянии сверхпроводимости, сопротивление равно нулю, поэтому, ток продолжает течь по цилиндрам. Этот ток создает магнитное поле, которое и вызывает силы отталкивания между цилиндрами и магнитом. Если же теперь мы поместим над цилиндрами магнит в виде колесика и раскрутим его, то он будет продолжать крутиться до тех пор, пока цилиндры находятся в состоянии сверхпроводимости. Заметим, что магнит крутится, не касаясь цилиндров и не нагреваясь, то есть никаких потерь энергии не происходит. Тем не менее, нет возможности получить сколь угодно большой ток в сверхпроводниках, поскольку определенное критическое значение силы тока разрушает состояние сверхпроводимости. Однако, конструкции, основанные на подобном принципе, могли бы существенно усовершенствовать электродвигатели и генераторы, значительно упростить устройства для аккумулирования энергии и многое другое. Поэтому, сегодня получение сверхпроводимости при комнатных температурах является одной из очень важных задач в физике.

Electrical resistance

http://en.wikipedia.org  Wikipedia, свободная энциклопедия

(везде ниже под сопротивлением понимается активное (резистивное) сопротивление, в котором происходит диссипация (рассеяние) электрической энергии и необратимый переход её в другие виды энергии, например, тепловую)

Электрическое сопротивление – физическая величина, характеризующая степень противодействия объекта электрическому току протекающему через него. Единица электрического сопротивления в системе СИ – ом ( 1 Ом). Величина, обратная электрическому сопротивлению, называется электрической проводимостью. Единица электрической проводимости в системе СИ – сименс (1 См). Механическим аналогом электрического сопротивления является трение.

Сопротивление объекта определяет отношение напряжения, приложенному к объекту, к току, протекающему через объект.

где       

             R  это сопротивление объекта, измеряется в омах, размерность Дж·с/Кл2

             V  это напряжение, приложенное к объекту, измеряется вольтах

             I   это ток, текущий через объект, измеряется в амперах

Для широкого ряда материалов и условий электрическое сопротивление не зависит от тока, протекающего через объект, и не зависит от напряжения, приложенного к объекту. Это значит, что сопротивление R постоянно.

Потери в электрическом сопротивлении

Когда ток I протекает через проводник, обладающий сопротивлением R, электрическая энергия преобразуется в тепловую энергию согласно закону Джоуля-Ленца. Мощность теплового потока:

где

         P  это мощность, измеряется в ваттах

         I   это ток, измеряется в амперах

         R  это сопротивление, измеряется в омах

Это явление преобразования энергии очень важно и используется в лампах накаливания и  электрических нагревателях. В других областях применения явление преобразования энергии вредно, так как вызывает потери энергии, например, в линиях передачи электроэнергии. В идеальном случае, проводники, служащие для соединений электрических устройств друг с другом, должны иметь нулевое сопротивление, но в реальных условиях только сверхпроводники отвечают этому требованию, тоесть являются идеальными проводниками. Общими способами уменьшения энергетических потерь в проводниках являются использование более толстых проводов и увеличение напряжений.

Сопротивление проводника

Сопротивление в цепи постоянного тока

Так как плотность тока постоянна в однородном проводнике, то сопротивление R проводника, через который течёт постоянный ток, может быть вычислено как

где

         l   это длина проводника, измеряется в метрах

         это площадь поперечного сечения проводника, измеряется в квадратных метрах  

         ρ  (Греческое: ро)  это удельное сопротивление материала проводника, измеряется в ом · метр. Удельное сопротивление характеризует способность оказывать противодействие электрическому току для данного материала проводника.

По практическим причинам почти любые подключения к реальному проводнику, с высокой вероятностью приведут к тому, что плотность тока в проводнике не будет постоянна. Тем не менее, эта формула даёт хорошую точность для длинных тонких проводников, таких, как проволочные провода.

Сопротивление в цепи переменного тока

(Для проводника, имеющего сопротивление R и  включенного в цепь переменного тока, так же выполняются законы Джоуля-Ленца и Ома; при этом используются среднеквадратичные значения тока и напряжения. Среднеквадратичное значение называется также эффективным или действующим. В случае переменного тока активное сопротивление является частью общего сопротитвления – импеданса. Импедансом называется полное (комплексное) сопротивление цепи переменного тока. Активное сопротивление это действительная часть импеданса.)

Если по проводу течёт переменный ток высокой частоты, то эффективная площадь сечения провода уменьшается по причине скин эффекта. Это приводит к увеличению сопротивления провода 10 dB на декаду, для провода, имеющего радиус много больший, чем глубина проникновения тока.

В проводнике, расположенном вблизи других проводников, фактическое сопротивление больше, чем предсказывает теория скин-эффекта, по причине эффекта близости.

Причины сопротивления

Сопротивление металлов

Металл состоит из атомной кристаллической решётки; атомы окружены электронными оболочками. Атомная решётка также известна, как положительная ионная решётка. Внешние электроны свободно отделяются от родительских атомов и путешествуют в кристаллической решётке, создавая «море» электронов, которое и делает металл проводником. Когда разность электрических потенциалов (тоесть напряжение, измеряемое в вольтах) приложено к металлу, электроны дрейфуют от одного конца проводника к другому под действием электрического поля.

При комнатной температуре тепловые колебания ионов решётки являются основным источником рассеивания электронов (благодаря ослабляющей интерференции свободной электронной волны на противоположных потенциалах ионов) – это основной случай металлического сопротивления. Неоднородности и дефекты кристаллической решётки также вносят вклад в сопротивление, хотя их вклад (в случае чистых металлов) пренебрежимо мал.

С увеличением площади поперечного сечения проводника большее число электронов становится проводниками тока, поэтому сопротивление уменьшается. С увеличением длины проводника увеличивается число случаев рассеяния на пути каждого электрона через металл проводника, поэтому сопротивление увеличивается. Сопротивление так же зависит от выбора металла.

Сопротивление полупроводников и изоляторов (диэлектриков)

(Уровень Ферми – наивысший энергетический уровень, занятый электронами при T = 0 K.)

В металлах уровень Ферми лежит в зоне проводимости, заполненной лишь частично. Электроны, находящиеся в зоне проводимости, получив сколь угодно малую энергетическую добавку (например, за счёт теплового движения или электрического поля), могут перейти на более высокий (свободный) энергетический уровень той же зоны, тоесть стать свободными электронами и участвовать в проводимости. С ростом температуры сопротивление будет расти, так как увеличивается рассеяние электронов проводимости на тепловых колебаниях решётки, и средняя длина свободного пробега электрона уменьшается.

В отличие от металлов, в химически чистых полупроводниках (так же называемых собственными) уровень Ферми лежит в середине запрещённой зоны, и расположен строго между нижним уровнем зоны проводимости и верхним уровнем валентной зоны. Это значит, что при температуре 0 Кельвин, валентная зона полностью заполнена электронами, а зона проводимости пустая, тоесть свободных электронов проводимости нет, и сопротивление стремится к бесконечности. Поэтому, при абсолютном нуле температур, чистые полупроводники не проводят электрического тока, тоесть являются изоляторами. С ростом температуры сопротивление будет уменьшаться из-за увеличения концентрации носителей заряда (электронов в зоне проводимости и дырок в валентной зоне).

В полупроводниках с примесной проводимостью примесные атомы увеличивают общее число свободных носителей заряда (создавая добавочные энергетические уровни в запрещённой зоне, тем самым расширяя валентную зону или зону проводимости). Донорные примеси увеличивают число электронов в зоне проводимости. Акцепторные примеси увеличивают число дырок в валентной зоне. И для донорных и для акцепторных примесных атомов увеличение концентрации примесей ведёт к уменьшению сопротивления. При очень высокой концентрации примесей полупроводник приближается по свойствам к металлу. При очень высоких температурах увеличивается число свободных носителей заряда, возбуждённых вследствие теплового движения, их вклад начинает преобладать над вкладом со стороны примесных атомов, и сопротивление будет уменьшаться с увеличением температуры по экспоненциальному закону. Поэтому, при высоких температурах, примесный полупроводник приближается по свойствам к собственному полупроводнику.

Сопротивление электролитов

Электролитами называются жидкие и твёрдые вещества, в которых перенос зарядов под действием электрического поля осуществляется не электронами или дырками, а положительными и отрицательными ионами, движущимися в противоположных направлениях. (Ион – электрически заряженная частица, образующаяся при отрыве или присоединении одного или нескольких электронов (или других заряженных частиц) к атому, молекуле, радикалу и другому иону.) К ионным проводникам, которыми являются электролиты, применимы законы Ома и Джоуля-Ленца (для растворов – за исключением областей сильных электрических полей и высоких частот). Сопротивление очень сильно зависит от свойств электролита и обычно значительно выше, чем у электронных проводников. Биологическим примером являются соли, проводящие ток в биологических мембранах.

Сопротивление различных материалов

Материал

Удельное сопротивление

ρ, Ом·м

Металлы

10 -8…10 -6

Полупроводники

10 -5…10 4

Электролиты

10 -2…10 8

Изоляторы

10 8 …10 18

Зонная теория

Применение законов квантовой механики показывает, что энергия электрона в атоме не может принимать произвольные значения. Вместо этого имеется определённое число фиксированных энергетических уровней, которые и может занять электрон, а междууровневые значения энергий — невозможны. Энергетические уровни группируются в зоны. Зон много, но в явлениях электропроводности участвуют только две зоны: валентная зона и зона проводимости. Зона проводимости расположена над валентной зоной. Электроны, находящиеся в зоне проводимости, могут передвигаться свободно через вещество, под действием электрического поля. 

В изоляторах и полупроводниках, атомы в веществе взаимодействуют таким образом, что между валентной зоной и зоной проводимости существует запрещённая зона, состоящая из множества запрещённых энергетических уровней, которые электрон занять не может. Поэтому, для протекания электрического тока, необходимы большие затраты энергии для того, чтобы электрон смог преодолеть запрещённую зону и достигнуть зоны проводимости. Поэтому, даже большие напряжения приводят к очень незначительным токам.

Дифференциальное сопротивление

В случае, когда сопротивление зависит от тока и напряжения, вводится понятие дифференциального сопротивления. Дифференциальное (возрастающее или убывающее) сопротивление определяется, как наклон графика ВАХ (вольт амперной характеристики) в заданной точке:

Эта величина иногда называется просто сопротивление , хотя эти два определения эквивалентны только для омических элементов, таких как идеальный резистор. Если график ВАХ не монотонный (например имеет пик или провал), дифференциальное сопротивление при определённых значениях тока и напряжения будет отрицательным. Эта особенность известна под названием «отрицательное сопротивление», хотя более правильным названием было бы отрицательное дифференциальное сопротивление, так как абсолютное сопротивление U/I остаётся положительным. Дифференциальное сопротивление также называют динамическим сопротивлением. 

Зависимость сопротивления от температуры

При температурах, близких к комнатной температуре, электрическое сопротивление типичных металлов линейно зависит от температуры. При низких температурах (ниже температуры Дебая), сопротивление уменьшается пропорционально T 5 , благодаря рассеянию электронов на фононах. При ещё более низких температурах, близких к 0 К, доминирует механизм рассеяния электронов на электронах и др. процессы, и сопротивление уменьшается пропорционально T Z, где Z=2,3,4 для разных металлов ( например T 2 для Li при T=1…10K). С некоторой точки, примеси в металле будут играть доминирующую роль в определении сопротивления и приведут к насыщению, тоесть сопротивление будет стремиться к постоянному значению. Правило Матиссена гласит, что сумма вышеперечисленных температурных зависимостей сопротивления, обусловленных различными причинами, даёт общее сопротивление как функцию температуры,

где Rimp это температуронезависимое сопротивление, обусловленное примесями, и a, b, и c – коэффициенты которые зависят от свойств металла.

Электрическое сопротивление типичного собственного (без примесей) полупроводника уменьшается экспоненциально с ростом температуры:

Полупроводники с примесной проводимостью имеют более сложный температурный профиль. При увеличении температуры, начиная от абсолютного нуля, сопротивление круто уменьшается, так как носители заряда покидают донорные и акцепторные уровни (примесная проводимость). Дальше, с ростом температуры, после того, как большинство донорных и акцепторных уровней потеряют носителей заряда (истощение примесей), сопротивление начнёт медленно расти благодаря уменьшению подвижности носителей заряда (как в металле). При ещё более высоких температурах вклад свободных носителей заряда, возбуждённых вследствие теплового движения, начинает преобладать над вкладом примесных донорно-акцепторных носителей заряда, и сопротивление будет резко уменьшаться с увеличением температуры по экспоненциальному закону (собственная проводимость), тоесть примесный полупроводник ведёт себя, как собственный полупроводник.

Электрическое сопротивление электролитов и изоляторов сильно нелинейно, и закон Ома выполняется только для ограниченных значений токов и напряжений. Поэтому обобщённые уравнения не могут быть приведены.

Измерение сопротивления

Прибор для измерения сопротивления называется омметр. Обычные омметры не могут точно измерять низкие значения сопротивлений (доли Ом), потому что собственное сопротивление прибора (и другие причины) вызывает падение напряжения, которое интерферирует с падением напряжения на измеряемом сопротивлении. Более точные приборы для измерения низких сопротивлений используют четырёх-проводные щупы (Four-terminal sensing / 4T sensing). Для измерения больших сопротивлений применяют мегаомметры и тераомметры.

 

Температурная зависимость удельного сопротивления — Материалы исследования для IIT JEE


Удельное сопротивление

Удельное сопротивление известно как удельное электрическое сопротивление или объемное сопротивление. Его можно определить как внутреннее свойство данного материала, которое показывает, как он противодействует потоку тока. Его также можно определить как сопротивление проводника, имеющего единичную длину и единицу площади поперечного сечения. Таким образом, это не зависит от длины и площади поперечного сечения материала.Но сопротивление материала зависит от длины и площади поперечного сечения материала. Удельное сопротивление выражается как ρ = R A / L, где R — сопротивление в омах, A — площадь поперечного сечения в квадратных метрах, а L — длина в метрах. Единица измерения удельного сопротивления — омметр.


Температурная зависимость удельного сопротивления

Удельное сопротивление материалов зависит от температуры. ρ t = ρ 0 [1 + α (T — T 0 ) — это уравнение, которое показывает связь между температурой и удельным сопротивлением материала.В уравнении ρ 0 — удельное сопротивление при стандартной температуре, ρ t — удельное сопротивление при t 0 C, T 0 — эталонная температура, а α — температурный коэффициент удельного сопротивления.

Изменение удельного сопротивления проводников

Мы знаем, что ток — это движение свободных электронов от одного атома к другому при наличии разности потенциалов. В проводниках нет запрещенной зоны между зоной проводимости и валентной зоной.Во многих случаях обе полосы перекрывают друг друга. Валентные электроны слабо связаны с ядром в проводниках. Обычно металлы или проводники имеют низкую энергию ионизации и поэтому очень легко теряют электроны. При подаче электрического тока делокализованные электроны могут свободно перемещаться внутри структуры. Так бывает при нормальной температуре.

При повышении температуры колебания ионов металлов в решетчатой ​​структуре возрастают. Атомы начинают колебаться с большей амплитудой.Эти колебания, в свою очередь, вызывают частые столкновения между свободными электронами и другими электронами. Каждое столкновение истощает часть энергии свободных электронов и делает их неспособными двигаться. Таким образом, он ограничивает движение делокализованных электронов. Когда происходит столкновение, скорость дрейфа электронов уменьшается. Это означает, что удельное сопротивление металла увеличивается и, следовательно, ток в металле уменьшается. Увеличение удельного сопротивления означает, что проводимость материала снижается.

Говорят, что для металлов или проводников они имеют положительный температурный коэффициент. Значение α положительное. Для большинства металлов удельное сопротивление линейно увеличивается с повышением температуры в диапазоне 500 К. Примеры для положительного температурного коэффициента включают серебро, медь, золото и т. Д.

Температурная зависимость удельного сопротивления металлов


Изменение удельного сопротивления в полупроводниках

Кремний — полупроводник.В полупроводниках ширина запрещенной зоны между зоной проводимости и валентной зоной мала. При 0K валентная зона полностью заполнена, а зона проводимости может быть пустой. Но при приложении небольшого количества энергии электроны легко перемещаются в зону проводимости. Кремний — это пример полупроводника. В нормальных условиях кремний действует как плохой проводник. Каждый атом кремния связан с 4 другими атомами кремния. Связи между этими атомами представляют собой ковалентные связи, в которых электроны находятся в фиксированных позитонах.Таким образом, при 0K электроны не перемещаются внутри структуры решетки.

При повышении температуры запрещенная зона между двумя зонами становится очень меньше, и электроны перемещаются из валентной зоны в зону проводимости. Таким образом, некоторые электроны ковалентных связей между атомами Si могут свободно перемещаться внутри структуры. Это увеличивает проводимость материала. Увеличение проводимости означает уменьшение удельного сопротивления. Таким образом, когда температура в полупроводнике повышается, плотность носителей заряда также увеличивается, а удельное сопротивление уменьшается.Говорят, что полупроводники имеют отрицательный температурный коэффициент. Значит, значение температурного коэффициента удельного сопротивления α отрицательно.

Кривая нелинейная в широком диапазоне температур.

Температурная зависимость от удельного сопротивления для полупроводников


Изменение удельного сопротивления в изоляторах

В изоляторах велика запрещенная зона между зоной проводимости и валентной зоной.Валентная зона полностью заполнена электронами. Запрещенная щель между двумя зонами будет больше 3 eV. Таким образом, для перехода валентного электрона в зону проводимости требуется большое количество энергии. Алмаз — это пример изолятора. Здесь все валентные электроны участвуют в образовании ковалентной связи и проводимости не происходит. Электроны прочно связаны с ядром.

Когда температура повышается, атомы материала колеблются, и это заставляет валентные электроны, присутствующие в валентной зоне, переходить в зону проводимости.Это, в свою очередь, увеличивает проводимость материала. Когда проводимость материала увеличивается, это означает, что удельное сопротивление уменьшается, и поэтому ток увеличивается. Таким образом, некоторые изоляторы при комнатной температуре превращаются в проводники при высокой температуре. Для изоляторов они имеют отрицательный температурный коэффициент. Значит, значение температурного коэффициента удельного сопротивления α отрицательно.

Проводники и изоляторы

Сверхпроводники

Мы знаем, что когда электрический ток проходит по проводникам, некоторая энергия теряется в виде тепла.Количество потерь энергии зависит от сопротивления материала. В 1911 году некоторые ученые охладили образец ртути до 4,2 ° выше абсолютного нуля. Таким образом сопротивление материала изменилось до нуля. Так был открыт первый сверхпроводник. Таким образом, ученые обнаружили, что в некоторых случаях некоторые материалы не проявляют никакого сопротивления. Материалы с нулевым сопротивлением называются сверхпроводниками. При нулевом сопротивлении материалы проводят ток без потери энергии.Когда температура таких материалов снижается, свободные электроны перестают сталкиваться с положительными ионами, и, таким образом, сопротивление оказывается нулевым. Температура, при которой сопротивление падает до нуля, называется Critical Temperature .

Когда сверхпроводник помещается в магнитное поле, магнитное поле огибает материал, не позволяя магнитному полю проходить через них. Когда напряженность магнитного поля увеличивается, в определенный момент поле может проникать через сверхпроводник и, таким образом, его поведение нарушается.

Считайте, что через сверхпроводник проходит электрический ток. Предположим, что плотность тока увеличивается, при определенном значении плотности тока он теряет свою сверхпроводимость и, наконец, ведет себя как нормальный материал. Плотность тока, выше которой материал теряет сверхпроводимость, называется критической плотностью тока. Высокая температура, сильное магнитное поле и высокая плотность тока разрушают сверхпроводимость материала. Сейчас эти материалы используются в аппаратах МРТ.

Прочие материалы

Удельное сопротивление таких материалов, как нихром, манганин и константан, не сильно зависит от температуры и показывает очень низкую зависимость. Следовательно, эти материалы используются в проволочных стандартных резисторах, поскольку изменение значения сопротивления незначительно при изменении температуры.

Манганин Константан


Факторы, влияющие на удельное сопротивление

Мы знаем, что удельное сопротивление ρ = m / ne 2 ԏ, где e — заряд электрона, ԏ — среднее время между столкновениями или время релаксации электронов, а m — масса электрона, n — плотность заряда.Таким образом, это показывает, что сопротивление зависит от ряда факторов, таких как время релаксации между столкновениями и плотность заряда. Из приведенных выше сценариев ясно, что при повышении температуры средняя скорость электронов увеличивается, и, следовательно, происходит больше столкновений. Таким образом, время релаксации между каждым столкновением уменьшается.

В случае металлов плотность заряда до некоторой степени не зависит от температуры. Таким образом, это влияет на другие факторы, такие как ԏ, что означает, что при повышении температуры среднее время между столкновениями уменьшается, что приводит к увеличению удельного сопротивления.

Для полупроводников и изоляторов плотность заряда n увеличивается при повышении температуры. Компенсирует уменьшение значения ԏ. Следовательно, удельное сопротивление уменьшается при понижении температуры.

Сводка
  • Удельное сопротивление — это сопротивление проводника, имеющего единицу длины и площади поперечного сечения. Единица измерения удельного сопротивления — омметр. Формула: ρ = RA / L, где R — сопротивление в омах, A — площадь поперечного сечения в квадратных метрах, а L — длина в метрах.

  • ρ t = ρ 0 [1 + α (T — T 0 ) — это уравнение, которое показывает связь между температурой и удельным сопротивлением материала. ρ 0 — удельное сопротивление при стандартной температуре, ρ t — удельное сопротивление при t 0 C, T 0 — эталонная температура, а α — температурный коэффициент удельного сопротивления.

  • Для металлов или проводников, когда температура увеличивается и удельное сопротивление металла увеличивается, и, таким образом, ток в металле уменьшается.У них положительный температурный коэффициент. Значение α положительное.

  • Для полупроводников, когда температура повышается, увеличивается проводимость материала. Это означает, что удельное сопротивление материала уменьшается, и поэтому ток увеличивается. Для полупроводников они имеют отрицательный температурный коэффициент. Значит, значение температурного коэффициента удельного сопротивления α отрицательно.

  • Для изоляторов электропроводность материала увеличивается при повышении температуры.Когда проводимость материала увеличивается, мы знаем, что удельное сопротивление уменьшается и, таким образом, увеличивается ток. Таким образом, некоторые изоляторы при комнатной температуре превращаются в проводники при высокой температуре. Для изоляторов они имеют отрицательный температурный коэффициент. Значение температурного коэффициента удельного сопротивления α отрицательно.

  • Материалы с нулевым сопротивлением называются сверхпроводниками. Температура, при которой сопротивление падает до нуля, называется критической температурой.Высокая температура, сильное магнитное поле и высокая плотность тока ослабят свойство сверхпроводимости материала. Меркурий — пример сверхпроводника.

  • Такие материалы, как нихром, манганин и константан, не сильно зависят от температуры. Таким образом, изменение удельного сопротивления материала при изменении температуры незначительно.


Посмотрите это видео, чтобы получить дополнительную информацию


Другие показания

Температурная зависимость удельного сопротивления

сайт cbse для класса 9 | Онлайн-классы CBSE | CBSE онлайн-класс 12

Температурное сопротивление — сопротивление различных материалов

Что такое сопротивление?

Сопротивление — это препятствие для потока электронов в материале.Когда к проводнику прикладывается разность потенциалов, это способствует движению электронов, в то время как сопротивление препятствует движению электронов. Комбинация этих двух факторов — это скорость, с которой заряд проходит между двумя терминалами.

Когда к веществу прикладывается напряжение, возникает электрический ток. Напряжение, приложенное к веществу, прямо пропорционально току.

V∝I

Константа пропорциональности называется удельным сопротивлением металлов.

В = RI

Следовательно, сопротивление определяется как отношение напряжения, приложенного к веществу, к току. Сопротивление измеряется в омах (Ом).

Единица сопротивления

Из концепции сопротивления можно сказать, что единицей электрического сопротивления является вольт на ампер. Одна единица сопротивления — это сопротивление, которое позволяет одной единице тока проходить через себя, когда к ней приложена одна единица разности потенциалов. Единица измерения сопротивления — вольт на ампер — называется ом (Ом).

Сопротивление различных материалов

  1. Проводники: те материалы, которые обладают очень низким сопротивлением потоку электронов. Серебро является хорошим проводником электричества, но из-за его высокой стоимости обычно не используется в электрических системах. Алюминий является хорошим проводником и широко используется в качестве проводника из-за его низкой стоимости и доступности.

  2. Полупроводники: Материал с умеренным значением сопротивления (не очень высоким и не очень низким) при комнатной температуре, известен как полупроводники.Есть несколько применений полупроводников, например, для изготовления электронных устройств. Кремний и германий — два материала, которые в основном используются для изготовления полупроводников.

  3. Изоляторы: те материалы, которые обладают очень высоким сопротивлением потоку электронов. Эти материалы являются очень плохими проводниками электричества и в основном используются в электрических системах для предотвращения утечки тока. Слюда, фарфор, бумага, сухое дерево, минеральное масло, газообразный азот, воздух и т. Д. Являются хорошими примерами изоляторов.

Сопротивление в зависимости от температуры

Общее правило гласит, что сопротивление в проводниках увеличивается с увеличением температуры и уменьшается с увеличением температуры в изоляторах.В случае полупроводников, как правило, сопротивление полупроводника уменьшается с повышением температуры. Но не существует простого математического соотношения для описания этой зависимости между сопротивлением и температурой для различных материалов с помощью графиков.

  1. Для проводника: валентная зона и зона проводимости перекрываются друг с другом в случае проводника. Итак, зона проводимости содержит избыточные электроны. Поглощая энергию, при повышении температуры больше электронов перейдет из валентной зоны в зону проводимости.

(Изображение будет добавлено в ближайшее время)

  1. Для полупроводников: проводимость полупроводникового материала увеличивается с увеличением температуры. При повышении температуры наиболее удаленные электроны приобретают энергию и, таким образом, приобретая энергию, наиболее удаленные электроны покидают оболочку атома.

(Изображение будет добавлено в ближайшее время)

Что такое удельное сопротивление?

Удельное сопротивление — это в основном количественное значение сопротивления, предлагаемого любым материалом.Хотя материалы сопротивляются прохождению электрического тока, одни из них лучше других проводят его. Удельное сопротивление — это показатель, позволяющий сравнивать, насколько разные материалы позволяют или сопротивляются току.

Единица измерения удельного сопротивления в системе СИ — омметр (Ом · м), обычно обозначаемый греческой буквой ρ, ро.

Удельное сопротивление материала можно определить как сопротивление (R), длину (L) и площадь материала (A).

ρ = RA / L

Из уравнения видно, что сопротивление можно изменять, регулируя ряд параметров.

Зависимость сопротивления от температуры

Удельное сопротивление материалов зависит от температуры как ρt = ρ0 [1 + α (T — T0). Это уравнение, которое показывает взаимосвязь между удельным сопротивлением и температурой.

ρt = ρ0 [1 + α (T — T0)

  • ρ0 — удельное сопротивление при стандартной температуре

  • ρt — удельное сопротивление при t0 C

  • T0 — эталонная температура

  • α — температурный коэффициент удельного сопротивления

Вот зависимость между удельным сопротивлением и температурой с графиками.

  1. Для проводников: Говорят, что проводники имеют положительную сопутствующую температуру, эффективную для металлов или проводников. Положительное значение — α. Для большинства металлов удельное сопротивление линейно увеличивается с увеличением температуры примерно на 500 К.

(Изображение будет добавлено в ближайшее время)

  1. Для полупроводников: удельное сопротивление полупроводника уменьшается с повышением температуры. Говорят, что у них отрицательный температурный коэффициент. Следовательно, температурный коэффициент удельного сопротивления α отрицателен.

(Изображение будет добавлено в ближайшее время)

  1. Изоляторы: для изоляторов при повышении температуры увеличивается проводимость материала. Когда проводимость материала увеличивается, мы знаем, что удельное сопротивление уменьшается, и, следовательно, увеличивается ток. А некоторые изоляторы превращаются в проводники при высоких температурах при комнатной температуре. У них отрицательный температурный коэффициент.

Интересные факты

Произошла ошибка при настройке пользовательского файла cookie

Этот сайт использует файлы cookie для повышения производительности.Если ваш браузер не принимает файлы cookie, вы не можете просматривать этот сайт.


Настройка вашего браузера для приема файлов cookie

Существует множество причин, по которым cookie не может быть установлен правильно. Ниже приведены наиболее частые причины:

  • В вашем браузере отключены файлы cookie. Вам необходимо сбросить настройки своего браузера, чтобы он принимал файлы cookie, или чтобы спросить вас, хотите ли вы принимать файлы cookie.
  • Ваш браузер спрашивает вас, хотите ли вы принимать файлы cookie, и вы отказались.Чтобы принять файлы cookie с этого сайта, используйте кнопку «Назад» и примите файлы cookie.
  • Ваш браузер не поддерживает файлы cookie. Если вы подозреваете это, попробуйте другой браузер.
  • Дата на вашем компьютере в прошлом. Если часы вашего компьютера показывают дату до 1 января 1970 г., браузер автоматически забудет файл cookie. Чтобы исправить это, установите правильное время и дату на своем компьютере.
  • Вы установили приложение, которое отслеживает или блокирует установку файлов cookie.Вы должны отключить приложение при входе в систему или проконсультироваться с системным администратором.

Почему этому сайту требуются файлы cookie?

Этот сайт использует файлы cookie для повышения производительности, запоминая, что вы вошли в систему, когда переходите со страницы на страницу. Чтобы предоставить доступ без файлов cookie потребует, чтобы сайт создавал новый сеанс для каждой посещаемой страницы, что замедляет работу системы до неприемлемого уровня.


Что сохраняется в файле cookie?

Этот сайт не хранит ничего, кроме автоматически сгенерированного идентификатора сеанса в cookie; никакая другая информация не фиксируется.

Как правило, в файлах cookie может храниться только информация, которую вы предоставляете, или выбор, который вы делаете при посещении веб-сайта. Например, сайт не может определить ваше имя электронной почты, пока вы не введете его. Разрешение веб-сайту создавать файлы cookie не дает этому или любому другому сайту доступа к остальной части вашего компьютера, и только сайт, который создал файл cookie, может его прочитать.

Влияние температуры на сопротивление

Влияние температуры на сопротивление

Электрическое сопротивление изменяется с изменением температуры.Сопротивление не только увеличивается с повышением температуры, но и уменьшается в некоторых случаях. Фактически, для разных типов материалов величина изменения сопротивления из-за изменения температуры различается, что обсуждается ниже.

Металл : Сопротивление всех чистых металлов линейно увеличивается с повышением температуры в ограниченном диапазоне температур. При низкой температуре ионы почти неподвижны. С повышением температуры ионы внутри металла приобретают энергию и начинают колебаться вокруг своего среднего положения.Эти колеблющиеся ионы сталкиваются с электронами. Следовательно, сопротивление увеличивается с повышением температуры.

Сопротивление всех металлов, таких как вольфрам, медь, алюминий и т. Д., Линейно увеличивается с повышением температуры в ограниченном диапазоне температур. Например, сопротивление меди составляет 100 Ом при 0 ° C, затем оно линейно увеличивается до 100 ° C. При температуре -234,5 ° C сопротивление меди практически равно нулю, как показано на рисунке.

Следовательно, чистый металл имеет положительный временный коэффициент сопротивления.

Сплав: Сопротивление почти всех сплавов увеличивается с повышением температуры, но скорость изменения сопротивления меньше, чем у металлов. Фактически, сопротивление некоторых сплавов, таких как манганин, эврика и константан, практически не изменяет сопротивления в значительном диапазоне температур. Благодаря этому свойству сплав используется для изготовления бокса сопротивления.

Полупроводник, изолятор и электролит: Сопротивление полупроводника, изолятора и электролита (кремний, стекло, лак и т. Д.) Уменьшается с увеличением температуры.При нулевой температуре полупроводник ведет себя как идеальный изолятор. При повышении температуры некоторые электроны приобретают энергию и становятся свободными для проводимости. Следовательно, с увеличением температуры увеличивается проводимость и уменьшается сопротивление.

Полупроводник имеет отрицательную временную составляющую удельного сопротивления, поэтому, поскольку с увеличением температуры сопротивление уменьшается.

Температурный коэффициент сопротивления:

Изменение сопротивления материала с повышением температуры может быть выражено через температурный коэффициент сопротивления.Рассмотрим проводник с сопротивлением R o при 0 ° c и R t при t ° c . Из приведенного выше обсуждения мы можем сделать вывод, что изменение сопротивления, т.е. (R t — R o ) равно

  1. , прямо пропорционально начальному сопротивлению R o
  2. Прямо пропорционально Повышение температуры t ° c .
  3. Зависит от материала проводникового металла и сплава

Отсюда

(R t — R o ) ∝ R o t

(R t — R o ) = αR o t

R t = R o (1 + α o t)

Где α o является постоянным и называется температурным коэффициентом сопротивления при 0 ° c и его значение зависит от природы материала и температуры.

Влияние температуры на температурный коэффициент сопротивления

Пусть R t1 и R t2 будет сопротивлением проводника при t 1 ° c и t 2 ° c соответственно, и α 1 и α 2 — соответствующий температурный коэффициент. Пусть проводник нагревается от начальной температуры t 1 ° c до конечной температуры t 2 ° c .

R t2 = R t1 [1 + α t1 (t 2 — t 1 )] ——————– 1

Теперь тот же проводник охлаждается от т 2 ° c до t 1 ° c.

R t1 = R t2 [1 + α t2 (t 1 — t 2 )] ——————— 2

Подставляем уравнение 2 в уравнение 1

R t2 = R t2 [1 + α t2 (t 1 — t 2 )] [1 + α t1 (t 2 — t 1 )]

Или

1 = [1 + α t2 (t 1 — t 2 )] [1 + α t1 (t 2 — t 1 )]

= [1 — α t2 (t 2 — t 1 )] [1 + α t1 (t 2 — t 1 )]

Примечание: Если температура изменяется от 0 ° C до t ° C, затем

Влияние температуры на удельное сопротивление

Удельное сопротивление или удельное сопротивление материала зависит от температуры.Изменение температуры влияет на удельное сопротивление материала так же, как и на сопротивление. Удельное сопротивление металлов линейно увеличивается с повышением температуры. Пусть ρt 1 и ρt 2 — удельное сопротивление при температуре t 1 ° c и t 1 ° c соответственно. Пусть м — наклон линейной части кривой.

Отношение м / ρt 1 называется температурным коэффициентом удельного сопротивления при t 1 ° c и почти равно α 1.

ρ t2 = ρ t1 [1 + α t1 (t 2 — t 1 )]

Примечание: При изменении температуры от 0 ° C до t ° C тогда

ρ t = ρ o [1 + α o t]


Ques1. Кусок медного провода имеет сопротивление 50 Ом при 10 ° C. Какова максимальная рабочая температура, если сопротивление провода нужно увеличить на 20%? Предположим, что α при 10 ° C = 0.0041 ° С -1 .

Sol: — R 1 = 50 Ом

R 2 = 50 + 0,2 x 50 = 60 Ом

t 2 = Неизвестная температура, при которой R 2 будет 60 Ом

Начиная с

R t2 = R t1 [1 + α t1 (t 2 — t 1 )]

2 R 1 [1 + α 1 (t 2 — t 1 )]

60 = 50 [1 + 0,0041 (t 2 — 10)]

Вопрос 2. Определенная медная обмотка имеет сопротивление 100 Ом при комнатной температуре. если температурный коэффициент сопротивления меди при 0 ° C составляет 0,00428 / ° C, рассчитайте температуру сопротивления обмотки E, увеличенную до 50 ° C. Предположим, что комнатная температура составляет 25 ° C.

R 1 = 100 Ом

t 1 = 25 ° C

t 2 = 50 ° C

α o = 0,00428 / ° C

Сейчас

= 0,003866 / ° C

R 2 = R 1 [1 + α 1 (t 2 — t 1 )]

R 2 = 100 [1 + 0.003866 (50-25)]

= 109,6657 Ом

сообщить об этом объявлении

Температурная зависимость удельного сопротивления — объяснение, формулы, решенные примеры задач

Температурная зависимость удельного сопротивления

Удельное сопротивление материала зависит от температуры. Экспериментально установлено, что для широкого диапазона температур удельное сопротивление проводника увеличивается с повышением температуры согласно выражению,


, где ρ T — удельное сопротивление проводника при T o C , ρ o — удельное сопротивление проводника при некоторой эталонной температуре (обычно при 20 º C) и α — температурный коэффициент удельного сопротивления.Он определяется как отношение увеличения удельного сопротивления на градус повышения температуры к удельному сопротивлению при T º.

Из уравнения (2.27) мы можем записать


, где ρ = ρ T ρ o — изменение удельного сопротивления при изменении температуры ∆T = Т — К. Единица измерения — градус по Цельсию.

α проводников

Для проводов α положительный. Если температура проводника увеличивается, средняя кинетическая энергия электронов в проводнике увеличивается.Это приводит к более частым столкновениям и, следовательно, к увеличению удельного сопротивления. График уравнения (2.27) показан на рис. 2.13

Несмотря на то, что удельное сопротивление проводников, таких как металлы, изменяется линейно в широком диапазоне температур, существует также нелинейная область при очень низких температурах. Удельное сопротивление приближается к некоторому конечному значению, когда температура приближается к абсолютному нулю, как показано на Рисунке 2.13 (b).

Поскольку сопротивление прямо пропорционально удельному сопротивлению материала, мы также можем записать сопротивление проводника при температуре T ºC как



Температурный коэффициент также можно получить из уравнения (2.28),


где R = RT −R — изменение сопротивления при изменении температуры ΔT = T –T

α полупроводников

Для полупроводников удельное сопротивление уменьшается с увеличением температуры. По мере повышения температуры из их атомов будет высвобождаться больше электронов (см. Блок 9 для информации о проводимости в полупроводниках). Следовательно, ток увеличивается и, следовательно, удельное сопротивление уменьшается, как показано на Рисунке 2.14. Полупроводник с отрицательным температурным коэффициентом сопротивления называется термистором.

Типичные значения температурных коэффициентов различных материалов приведены в таблице 2.3.


Температурную зависимость удельного сопротивления можно понять следующим образом. В разделе 2.1.3 мы показали, что электрическая проводимость σ = ne 2τ / m . Поскольку удельное сопротивление м обратно пропорционально σ, его можно записать как:


Удельное сопротивление материалов

i) обратно пропорционально количественной плотности (n) электронов

ii) обратно пропорционально среднее время между столкновениями (τ).

В металлах, если температура увеличивается, среднее время между столкновениями (τ) уменьшается, и n не зависит от температуры. В полупроводниках при повышении температуры n увеличивается, а τ уменьшается, но увеличение n является преобладающим, чем уменьшение τ, так что общее удельное сопротивление уменьшается.


ПРИМЕР 2.13

Если сопротивление катушки составляет 3 Ом при 20 ° C и α = 0,004 / 0C, определите ее сопротивление при 100 ° C.

Раствор

R0 = 3 Ом, T = 100ºC, T0 = 20ºC

α = 0.004 / ºC, RT =?

RT = R0 (1 + α (T-T0))

R100 = 3 (1 + 0,004 × 80)

R100 = 3 (1 + 0,32)

R100 = 3 (1,32)

R100 = 3,96 Ом

ПРИМЕР 2.14

Сопротивление материала при 10 ° C и 40 ° C составляет 45 Ом и 85 Ом соответственно. Найдите его температурный коэффициент сопротивления.

Раствор

T0 = 10 ° C, T = 40 ° C, R0 = 45 Ом, R = 85 Ом

α = 1 / R. ΔR / ΔT


α = 0.0296 по ºC

Влияние температуры на сопротивление

  • Изучив этот раздел, вы сможете:
  • • Опишите влияние температуры на сопротивление проводника.
  • • Опишите влияние температуры на сопротивление изолятора.
  • • Определите отрицательные и положительные температурные коэффициенты.

Как температура изменяет сопротивление

Хотя сопротивление проводника изменяется с размером проводника (например,грамм. более толстые провода имеют меньшее сопротивление току, чем более тонкие), сопротивление проводника также изменяется с изменением температуры. Этого можно ожидать, потому что при изменении температуры размеры проводника будут изменяться по мере того, как он расширяется или сжимается.

Однако материалы, которые классифицируются как ПРОВОДНИКИ, имеют тенденцию ПОВЫШАТЬ свое сопротивление с повышением температуры. Однако ИЗОЛЯТОРЫ могут УМЕНЬШИТЬ свое сопротивление с повышением температуры.Материалы, используемые для изготовления изоляторов (стекло, пластик и т. Д.), Демонстрируют заметное падение сопротивления только при очень высоких температурах. Они остаются хорошими изоляторами при любых температурах, с которыми они могут столкнуться при использовании.

Таким образом, эти изменения сопротивления нельзя объяснить изменением размеров из-за теплового расширения или сжатия. Фактически, для проводника данного размера изменение сопротивления в основном связано с изменением удельного сопротивления материала и вызвано изменением активности атомов, составляющих материал.

Температура и атомная структура

Причины этих изменений удельного сопротивления можно объяснить, рассмотрев протекание тока через материал. Поток тока — это фактически движение электронов от одного атома к другому под действием электрического поля. Электроны — это очень маленькие отрицательно заряженные частицы, которые отталкиваются отрицательным электрическим зарядом и притягиваются положительным электрическим зарядом. Следовательно, если к проводнику приложен электрический потенциал (положительный на одном конце, отрицательный на другом), электроны будут «мигрировать» от атома к атому к положительному полюсу.

Однако только некоторые электроны могут свободно перемещаться. Остальные внутри каждого атома так сильно привязаны к своему конкретному атому, что даже электрическое поле не сместит их. Следовательно, ток, протекающий в материале, обусловлен движением «свободных электронов», и количество свободных электронов в любом материале по сравнению с теми, которые тесно связаны с их атомами, определяет, является ли материал хорошим проводником (много свободных электронов) или хороший изолятор (почти нет свободных электронов).

Воздействие тепла на атомную структуру материала заставляет атомы колебаться, и чем выше температура, тем сильнее колеблются атомы.

В проводнике, по которому уже протекает большое количество свободных электронов, колебания атомов вызывают множество столкновений между свободными электронами и захваченными электронами. Каждое столкновение использует некоторую энергию свободного электрона и является основной причиной сопротивления. Чем больше атомы толкаются в материале, тем больше возникает столкновений и, следовательно, больше сопротивление току.

Однако в изоляторе ситуация несколько иная. Свободных электронов так мало, что ток почти не протекает. Почти все электроны прочно связаны внутри своего атома. Нагрев изолирующего материала вызывает вибрацию атомов, и при достаточном нагреве атомы вибрируют достаточно сильно, чтобы фактически высвободить часть своих захваченных электронов, создавая свободные электроны, которые становятся носителями тока. Поэтому при высоких температурах сопротивление изолятора может упасть, а в некоторых изоляционных материалах — довольно резко.

В материале, сопротивление которого УВЕЛИЧИВАЕТСЯ с повышением температуры, считается, что материал имеет ПОЛОЖИТЕЛЬНЫЙ ТЕМПЕРАТУРНЫЙ КОЭФФИЦИЕНТ.

Когда сопротивление ПАДАЕТ с повышением температуры, говорят, что материал имеет ОТРИЦАТЕЛЬНЫЙ ТЕМПЕРАТУРНЫЙ КОЭФФИЦИЕНТ.

Как правило, проводники имеют ПОЛОЖИТЕЛЬНЫЙ температурный коэффициент, в то время как (при высоких температурах) изоляторы имеют ОТРИЦАТЕЛЬНЫЙ температурный коэффициент.

Различные материалы в каждой группе имеют разные температурные коэффициенты.Материалы, выбранные для изготовления резисторов, используемых в электронных схемах, представляют собой тщательно отобранные проводники с очень низким положительным температурным коэффициентом. При использовании резисторы, изготовленные из таких материалов, будут иметь лишь очень небольшое увеличение удельного сопротивления и, следовательно, их сопротивления. Использование таких материалов для изготовления резисторов создает компоненты, значение которых незначительно изменяется в заданном диапазоне температур.

Материалы, выбранные в качестве изоляторов, будут иметь очень низкий ОТРИЦАТЕЛЬНЫЙ ТЕМПЕРАТУРНЫЙ КОЭФФИЦИЕНТ во всем рабочем диапазоне температур.

Удельное сопротивление Температурная зависимость: определение, единица измерения, температура

Температурная зависимость удельного сопротивления

Удельное сопротивление материалов зависит от температуры. ρt = ρ0 [1 + α (T — T0) — это уравнение, которое показывает связь между температурой и удельным сопротивлением для любого материала. В этом уравнении ρ0 — удельное сопротивление материала при определенной температуре, ρt — удельное сопротивление материала при t o ° C, T0 — эталонная температура или температура, при которой будут проводиться измерения, а α — температурный коэффициент удельного сопротивления.

Удельное сопротивление косвенно пропорционально температуре. С повышением температуры материалов удельное сопротивление будет уменьшаться. Но это не всегда верно для всех материалов, т.е. не все материалы имеют одинаковые изменения температуры.

Удельное сопротивление металлических проводников находится в ограниченном диапазоне температур, который определяется следующим уравнением

ρT = ρ0 [1 + a (T – T0)]

Здесь

ρT = удельное сопротивление при температуре T

ρ0 = удельное сопротивление при эталонной температуре T0

a = температурный коэффициент удельного сопротивления материала, значение a равно (Температура) -1

В соответствии с приведенным выше уравнением график зависимости ρT от T будет прямой линией, что означает, что удельное сопротивление металлического проводника увеличивается с увеличением температуры.У разных материалов разные температурные колебания. Например, такие материалы, как нихром, хром и константан, очень мало изменяют свое удельное сопротивление с температурой. Следовательно, они используются в резисторах с проволочной обмоткой. С другой стороны, полупроводники показывают обратную зависимость от температуры. Удельное сопротивление полупроводников уменьшается с повышением температуры.

В терминах «n»

Мы знаем удельное сопротивление, ρ определяется как

ρ = 1 / σ = m / ne 2 ζ

Здесь n — номер.свободных электронов в материале, а ζ — это среднее время между столкновениями, которое обратно пропорционально ρ.

In «n» в металле не изменяется с температурой. Однако с повышением температуры будет увеличиваться количество столкновений электронов. Это уменьшает ζ, что означает, что с повышением температуры ρ увеличивается. Однако в изоляторах и полупроводниках «n» увеличивается с повышением температуры. Таким образом, с повышением температуры «ρ» уменьшается.

Изменение удельного сопротивления

Удельное сопротивление ρ материала зависит от молекулярной и атомной структуры и зависит от температуры. У многих хороших проводников удельное сопротивление увеличивается с повышением температуры.

Изменение удельного сопротивления в проводниках

В проводниках валентные электроны слабо связаны с ядром. Металлы или проводники имеют относительно низкую энергию ионизации, поэтому металлы очень легко теряют электроны. Когда электрический ток подается на металл, делокализованные электроны свободно перемещаются внутри соединения.

Когда мы увеличиваем температуру, движение ионов металлов в структуре материала также увеличивается. Ведь металл начинает вибрировать с большей амплитудой. Эти движения, в свою очередь, вызывают несколько столкновений между свободными электронами и другими электронами. Каждое столкновение вызывает потерю некоторой энергии свободных электронов и лишает их свободного движения. Он ограничивает движение свободных электронов. Когда происходит столкновение, средняя скорость электронов уменьшается.Следовательно, удельное сопротивление металла увеличивается, и, следовательно, ток, протекающий в металле, уменьшается. Удельное сопротивление увеличивается, следовательно, проводимость материала уменьшается.

Изменение удельного сопротивления в полупроводниках

Кремний является примером полупроводников. В полупроводниках ширина запрещенной зоны между зоной проводимости и валентной зоной меньше, чем в чистом проводнике. При температуре 0K валентная зона заполнена, а зона проводимости может оставаться пустой или частично заполненной.

Рис: температура (T) изменчивость с удельным сопротивлением (ρ) представлен полупроводник

При подаче небольшого количества энергии электроны легко перемещаются в зона проводимости, которая увеличивает проводимость полупроводника. Кремний — это пример полупроводников. В нормальных условиях кремний действует как плохой проводник. Атом кремния связан с 4 другими атомами кремния в структуре решетки кремния.

Изменение удельного сопротивления в изоляторах

В изоляторах энергетическая щель между зоной проводимости и валентной зоной очень велика. В изоляторах валентная зона частично заполнена электронами. Запрещенный промежуток между двумя зонами будет больше 3 эВ, что требует энергии для перемещения электронов в зоны проводимости. Стекло, пластик и алмаз — примеры изолятора.

Что следует помнить

Когда мы говорим об удельном сопротивлении, оно зависит также от других факторов.Факторы, влияющие на удельное сопротивление электрических материалов:

  • Температура.
  • Легирование.
  • Механическое напряжение.
  • Возрастная закалка.
  • Холодная обработка.

Помните: при определении химических и физических характеристик каждого материала удельное сопротивление играет полезную роль при сравнении различных материалов на основе их способности проводить электрические токи. Высокое сопротивление означает плохие проводники. Следовательно, люди могут соответственно отдавать предпочтение материалам.

Примеры вопросов

Вопросов. Как вы объясните температурную зависимость удельного сопротивления полупроводника? (2 балла)

Отв. Для всех полупроводников удельное сопротивление уменьшается с повышением температуры. С повышением температуры больше электронов отделяется от своих атомов. Полупроводник с отрицательным температурным коэффициентом сопротивления называется термистором.

Вопрос. Зависит ли удельное сопротивление металла от температуры? (2 балла)

Отв. Удельное сопротивление обратно пропорционально температуре. Кроме того, при повышении температуры материалов их удельное сопротивление будет уменьшаться.

Вопрос. Меняется ли удельное сопротивление полупроводников с температурой? (2 балла)

Отв. По мере повышения температуры большее количество электронов будет получать энергию для перехода из зоны проводимости в валентную зону и, следовательно, увеличивает проводимость полупроводника. Таким образом, с повышением температуры удельное сопротивление полупроводников уменьшается.

Вопрос. Почему удельное сопротивление уменьшается с температурой? (2 балла)

Отв. Когда температура увеличивается, промежуток между зонами становится очень меньше, и электроны перемещаются из валентной зоны в зону проводимости. Таким образом, когда температура в полупроводнике увеличивается, плотность заряженных электронов также увеличивается, а удельное сопротивление уменьшается.

Вопрос. Какое влияние оказывает температура на полупроводники? (2 балла)

Отв. При повышении температуры ковалентные связи между соединениями разрушаются из-за тепловой энергии, подводимой к полупроводникам. Таким образом становятся свободными электроны, которые участвуют в образовании связей. Следовательно, высокотемпературный полупроводник больше не ведет себя как изолятор.

Вопрос. Как изменить удельное сопротивление? (2 балла)

Отв. Удельное сопротивление материалов зависит от атомной структуры материала. Таким образом, мы можем изменить удельное сопротивление вещества, изменив температуру.Мы также знаем, что валентные электроны слабо удерживаются в ядре.

Вопрос. Два провода одинаковой длины, один из меди, а другой из манганина, имеют одинаковое сопротивление. Какой провод толще? (2 балла)

Отв. Поскольку R = pl / A, следовательно, A = pl / R

Для обоих проводов R и l одинаковы, и ρ медь

∴ Медь <манганин

Таким образом, мы можем сказать, что манганиновая проволока толще, чем медная.

Вопрос. Провод с сопротивлением 20 Ом постепенно растягивается до удвоения исходной длины. Затем его разрезают на две равные части. Затем эти части подключаются параллельно через 4,0-вольтовую батарею. Найдите ток, потребляемый от батареи. (3 балла)

Отв. Когда мы растягиваем провод, его сопротивление увеличивается в четыре раза, то есть 80 Ом, поскольку объем постоянный, и R α l 2

Таким образом, две равные части будут иметь сопротивление 40 Ом каждая.

При параллельном подключении эквивалентное сопротивление будет 20 Ом.

Следовательно, потребляемый ток будет равен = V / R eq

Таким образом, он будет равен = 4/20 = 0,2A.

Вопрос. В чем разница между удельным сопротивлением и сопротивлением? (4 балла)

Отв. Разница между удельным сопротивлением и сопротивлением составляет:

Сопротивление

00 Определение 00 Материал 9010 мс 03 9010 мс метр

Параметр

Сопротивление

Сопротивление

Сопротивление ток

Сопротивление материала, имеющего определенные размеры

Символ

R

P

SI Unit03

Формула

R = V / I или,

R = ρ (L / A)

V = напряжение, I = ток, ρ = удельное сопротивление

ρ = (R × A) / L

R = сопротивление, L = длина, A = поперечное сечение ar ea

Зависимость

Температура, длина, площадь поперечного сечения проводника

Температура

Области применения

Предохранители, как элементы сопротивления, полезны в качестве элементов сопротивления.

Разное

Добавить комментарий

Ваш адрес email не будет опубликован. Обязательные поля помечены *